くだらねぇ問題はここへ書け ver.3.14(22桁略)3832

このエントリーをはてなブックマークに追加
1 ◆Ea.3.14dog
いちいちスレッド建てないで,ここに書いてね.

最重要な数学記号の書き方の例(これを読まないと放置される可能性大)
---------------------------------------------------------------

   ※分数は、分母分子がわかるように括弧を沢山使ってください。
    1+a/bでは1+(a/b),(1+a)/bの2通りの解釈ができます。
    その他解釈の仕方が幾通りもある例がたっぷりあるので気をつけてください。

    これを無視すると放置される可能性が大です。

--------------------------------------------

●足し算 a+b ●引き算 a-b ●掛け算 a*b, ab ●割り算・分数 a/b, a/(b+c), a/(b*c)
※“*”は掛け算の記号です。×(かける)はXx(エックス)と混同してしまうので使わないのが無難です。
※割り算は“÷”を使わず分数の形で表わすのが一般的です。
※分数は、分母分子がわかるように括弧を沢山使ってください。1+a/bでは1+(a/b),(1+a)/bの2通りの解釈ができます。
●指数 a^b, x^(n+1)
●ルート √(a+b), (a+b)^(1/2)
※指数は“^”を使います。「xのn+1乗」は“x^(n+1)”ときちんと括弧でくくりましょう。
※√は“るーと”を変換して下さい。
※さらに詳しい書き方、過去スレはhttp://members.tripod.co.jp/mathmathmath/にあります

前スレと関連スレは>>2-4
2 ◆Ea.3.14dog :03/08/03 04:03
【前スレと関連スレ】
くだらねぇ問題はここへ書け ver.3.14(21桁略)3383
http://science.2ch.net/test/read.cgi/math/1058382041/l50
雑談はここに書け!【12】
http://science.2ch.net/test/read.cgi/math/1059231618/l50

◆わからない問題は絵で書いて質問◆
http://science.2ch.net/test/read.cgi/math/1040698718/l50
   救済スレ   
http://science.2ch.net/test/read.cgi/math/1039581014/l50
よくある質問
http://www.geocities.co.jp/CollegeLife-Club/7442/math/index.html
3132人目の素数さん:03/08/03 07:28
>>1
4132人目の素数さん:03/08/03 08:23
次の条件を満たす(x,y,z)の組は何個あるか?

x+y+z=100
5≦x≦50
5≦y≦45
0≦z≦35

どなたかご教授ねがいます。
5132人目の素数さん:03/08/03 08:29
多すぎて書ききれないくらいある
6132人目の素数さん:03/08/03 08:30
>>4
解は存在しないんじゃない?
オレの勘違いか?
7132人目の素数さん:03/08/03 08:41
z=5とすると、
x+y=95⇒(50,45)
z=6とすると、
x+y=94⇒(49,45)(50,44)
以下同じように増えていく
z=35とすると、
x+y=65⇒(50,15)(49,16)・・・・・・(20,45)
このとき、31個できる。

よって、1+2+・・・・・・+30+31=496

であってるのではないかと思うんだけど違ったらスマソ
8132人目の素数さん:03/08/03 10:18
>>7
勝手に整数解にすんなよ
9高1 ◆KOU1ujLJwo :03/08/03 11:12
正四面体の各面に色を塗りたい。ただし、1つの面には1色しか塗らないものとし
色を塗ったとき、正四面体を回転させて一致する塗り方は同じとみなすことにする。

(1) 異なる3色の色がある場合を考える。3色すべて使うときは、その塗り方は
全部で何通りあるか。また、3色のうち使わない色があってもよいときは
その塗り方は全部で何通りあるか。


この問題が分からないので教えてください。
>>9
どういう塗り方があるか全て調べて数えるだけ。
11トップエリート街道さん ◆BIG6e4aEMg :03/08/03 12:55
まず4色あって4色を使う場合を考えるのがわかりやすいと思う。
色をA,B,C,Dとする。正四面体に色を塗ったとして、
Aの面が下になるように机の上に置く。
そうすると、あとは水平な回転しかできないわけだが、
Bの面を手前に向ける。
こうすると、もう回転のさせようがなくなる。
よって、C,Dを向こうの面の左右どちらに塗るかで、回転で一致しない2つの塗り方ができる。
 3色あって3色使う場合は、まず1つの色を2面に塗らなくてはならないから、
その選び方がC(3,1)=3通り。
Aを2面に塗ったとき、Bの面を下に、Cの面を手前にもってくると、
向こう側は2面ともAだから、どう塗っても回転で重ねられる。つまり1通り。
よって3色あって3色使う場合は、3×1=3(通り)
 あとは一度自分で考えてみてね。
12132人目の素数さん:03/08/03 12:58
(問)2次関数f(x)=x2乗-2ax+a-1(0≦x≦2)の最大値Mを求めなさい。
>>9
3色全てを使う場合、使う色はAABCのパターンになる。
この同色の隣り合った面(AA)に着目すると、
AABCで塗る場合は1通りしかないことがわかる。
(片手でAAの部分をつまんで持ち上げ、自分の方に
向けてみることを想像するとよい。)

従って3色全部使う場合は全部で
AA-B-C
BB-C-A
CC-A-B
の3通り。
>>12
区間の端点で最大値をとるからf(0)とf(2)の大小で場合分け
15132人目の素数さん:03/08/03 13:10
凸なn角形がある。この多角形の4つの頂点を結んで作る
四角形のうち、多角形と2つの辺だけ共有するものはいくつあるか?

1つの辺だけを共有するものはn-{(n-5)C2)}個とわかったのですが、
2辺の場合がわかりません。どなたかお教えください。
地球科学関係の学生です。

 実験データーにたいして、横軸が対数軸のガウス関数でフィッティングしているのですが、求めたフィッティング結果の面積を求められなくて困っています。

具体的には、


g(x) = ( N / (√(2π)*ρ) ) * exp( - ( ( log(x) - log(c) )^2 ) / (2*ρ^2) )

の時、

∫[-∞,∞] g(x) dx

を求めたいです。

普通のガウス関数でしたら、極座標変換して解けることはわかったのですが、x軸が対数だと私の力では解くことができませんでした。


よろしくお願いします。
17トップエリート街道さん ◆BIG6e4aEMg :03/08/03 13:23
>>15
>n-{(n-5)C2)}個

2辺だけ共有する時は、それが隣り合う2辺か、向かい合う2辺かで場合わけ。
18132人目の素数さん:03/08/03 13:46
>>17
ありがとうございます。ただ、向かい合わない&隣り合わないときも
条件が成り立つときがあるとおもうのですが…
どなたか、ご教示ください。
19132人目の素数さん:03/08/03 13:52
余弦・正弦を使ったsin cosの加法定理の証明を
教えて下さい。宜しくお願いします。
20132人目の素数さん:03/08/03 13:53
>>19
余弦 = cos
正弦 = sin

で何を言いたいのか・・・?
21132人目の素数さん:03/08/03 13:55
>>19
普通に参考書みればわかるはず。
22トップエリート街道さん ◆BIG6e4aEMg :03/08/03 14:00
>>16 計算は自信ないけど
積分区間は(0,∞)だよね?
N / (√(2π)*ρ)=aとする。
ρ>0として、(log(x)-log(c))/(√2・ρ)=tとおくと、
log(x)-log(c)=log(x/c) =√2・ρt
x=cexp(√2・ρt)
dx=√2・cρexp(√2・ρt)dt で、
∫[0,∞] g(x) dx =∫[-∞,∞] aexp(-t^2)√2・cρexp(√2・ρt)dt
=√2・acρ∫[-∞,∞] exp(-t^2+√2・ρt)dt
=√2・acρexp(ρ^2/2)∫[-∞,∞] exp(-(t-ρ/√2)^2)dt
=√2・acρexp(ρ^2/2)√π
=Ncexp(ρ^2/2)
もし∫[c,∞] g(x) dx ならこの半分
2319:03/08/03 14:01
すいません

cos(α+β)=cosαcosβ−sinαsinβ

を図を用いて証明せよ。という問題です。
24トップエリート街道さん ◆BIG6e4aEMg :03/08/03 14:12
>>23
O(0,0)として、角AOBがα+βになるようにA,Bをとるんだけど、どうとったら簡単になると思う?
25132人目の素数さん:03/08/03 14:12
3,3,8,8の4つの数字を全て使って24になるような
式を考えないといけないんですけど...
分かりますか?
26コピペでスマソ:03/08/03 14:15
ttp://that.2ch.net/test/read.cgi/gline/1059880142/
★ただいま2ちゃんねらー分布地図の製作を行っています
 お手数ですが、時間に余裕がありましたら
 本スレに都道府県と市町村名をカキコしてください
 現時点での分布はこの図のようになっています
 http://map2ch.tripod.co.jp/map.png

Part1 http://life2.2ch.net/test/read.cgi/kankon/1059631235/
Part2 http://life2.2ch.net/test/read.cgi/kankon/1059751025/
Part3 http://life2.2ch.net/test/read.cgi/kankon/1059842791/
Part4 http://life2.2ch.net/test/read.cgi/kankon/1059877937/

3日前に生活板に立ち、ものすごい勢いで伸びる「2ちゃんねらー分布地図」スレのガイドライン。
自分がどこに住んでいるかは本スレのほうに書いてください。
このスレでは集計、地図化および集計に関する議論などを行います。
>>25
8/(3-8/3)=24
29132人目の素数さん:03/08/03 15:27
>>17さん
失礼しました。n*{(n-5)C2}ですね。

う〜ん、2辺共有の場合を考えているんですけど、
わからないです。どなたかお助けを・・・
>>29
隣り合わない2辺を選んだ場合、頂点が4つ確定するが
隣り合う2辺を選んだ場合は、頂点が3つしか確定しないので
もう一つ頂点を選ぶ必要があるが3辺共有にならないように注意すればよし
3115:03/08/03 15:54
15ですが、
辺が隣り合う場合、(n-5)C1
辺が隣り合わない場合、(n-4)C1
となる。というので、あってますでしょうか?
たびたびすみません・・・
32132人目の素数さん:03/08/03 15:56
>>29
1辺だけ共有のほうが難しいと思うが・・・。n*{(n-5)C2}になるのかなあ?
2辺を共有は、隣接する2辺を共有する場合と向かい合う2辺を共有する場合とに分ける。
・隣接する2辺を共有する場合
隣接する2辺の取り方がn通り。もうひとつの頂点の取り方が(n-5)C1=n-5 通り。
あわせてn(n-5) 通り。
・向かい合う2辺を共有する場合
ある1辺の取り方がn通り。この辺に対して、向かい合う辺の取り方がn-3通り。
あわせてn(n-3)通り。
結局、全部で 2n(n-4) 通り。
>>32
そんなんじゃ全然だめだよ
何年生か知らないけど。
3432:03/08/03 16:07
スマソ・・・・
>向かい合う2辺を共有する場合
>ある1辺の取り方がn通り。この辺に対して、向かい合う辺の取り方がn-3通り。
>あわせてn(n-3)通り。
ここは間違い。
・向かい合う2辺を共有する場合
ある1辺の取り方がn通り。この辺に対して、向かい合う辺の取り方がn-5通り。
2つずつ重複して数えているのであわせてn(n-5)/2通り。
全部で、(3/2)*n(n-5) 通り。
3515:03/08/03 16:09
詳しいご説明どうもありがとうございました。>>32さん

ちなみに、1辺のときは、こう求めました。
多角形と1つの辺を共有するのだから、
1点がきまれば、もう1つの頂点が決定する。
また、その後は必ず1つはあけていかないといかないので、
(n-5)C2となる。
それぞれの頂点からなので、
n*{(n-5)C2}個となる。
>35
>また、その後は必ず1つはあけていかないといかないので、
>(n-5)C2となる。

????
37132人目の素数さん:03/08/03 16:35
うちに自動車がないので(単にとけないだけかも)、さっぱりわかりません。
ヒントでも構いませんので、
どなたかよろしくお願いいたします。

自動車の前後輪の幅をL、内外輪の幅をWとする。
また、この車が回転するときは、前輪のみが回転し、
後輪はつねにまっすぐである。
内側前輪、外側前輪の切れ角をα、βとし、
外側前輪の回転半径をRとする。

(1)α、βと、L、Wの間に成り立つ関係を調べよ。
(2)ある車では、L=2.715m、W=1.460mで、
ハンドルを最大に切ったときの最小回転半径がR=5.4mであった。
このときの、α、βのもとめかたは?
(車の最小回転半径とは、普通Rを指す)
(3)自動車を車庫に入れる場合、
前進と後退では、どちらのほうがいれやすいか?
>>37
まず自動車を買うことから始めようよ。。。
39_:03/08/03 16:47
40132人目の素数さん:03/08/03 17:08
(問)2|x|+|x-3|≦9
だれか、教えてください!
41132人目の素数さん:03/08/03 17:10
>>40
教科書嫁
42トップエリート街道さん ◆BIG6e4aEMg :03/08/03 17:18
>>37
それぞれの車輪の向きが同心円の接線になっていると思えばいいんじゃないかな?
接線と半径が垂直な事を使えば図が描ける。
後輪はまっすぐ向いてるから、後ろの車軸の延長上に中心が来る。あとは三角比の計算。
(3)は図をよく見て考えるといい。
>>40
x≧3のとき、この不等式は2x+x−3≦9⇔x≦4、すなわち3≦x≦4が解。
0≦x≦3のとき、この不等式は2x−x+3≦9⇔x≦6、すなわち0≦x≦3が解。
x≦0のとき、この不等式は−2x−x+3≦9⇔x≧−2、すなわち−2≦x≦0が解。
合わせて、x≧−2
>>43 書き間違えた。
× 合わせて、x≧−2
○ 合わせて、−2≦x≦4
45132人目の素数さん:03/08/03 17:25
今日のぼるじょあの中の人は低レベルだな
質問するのはやめておこう
46132人目の素数さん:03/08/03 17:33
>>43
なんで0と3が出てくるんですか?
絶対値記号の中身が0になるところ。
>>46
いきなり>>43みたいに解く前に、y=2|x|+|x−3|のグラフとy=9のグラフを書くといい。
そうすりゃ何故0と3が必要かわかる。
49132人目の素数さん:03/08/03 17:53
<>
↑の読み方はなんと読むのでしょうか?
どなたか教えてください。。。
5037:03/08/03 17:59
37の問題を42さんのヒントをもとに、といたのですが、
(1)(3)がそれぞれわかりません。
どなたか、もうちょっとヒントをくださいませんでしょうか?

>>49
>…大なり <…小なり ≦…小なりイコール
51132人目の素数さん:03/08/03 18:04
>>48
y=2|x|+|x-3|のグラフってどうやって書けばいいの?
x≧3のとき
0≦x≦3のとき
x≦0のとき
の3つに場合わけしてグラフを書く。
教科書・参考書読み直すことを勧める。
53トップエリート街道さん ◆BIG6e4aEMg :03/08/03 18:13
>>50
回転の中心をO, 内側前輪、外側前輪、内側後輪、外側後輪をそれぞれA, B, C, D
とすると、∠AOC=α, ∠BOC=βだよ。
(3)は、車庫は道路に垂直で、幅に余裕がない状況を考えていると思う。
道路に平行な車を道路に垂直に向けるには道幅がどれぐらい要るか、と考えるといい。
54_:03/08/03 18:18
55132人目の素数さん:03/08/03 18:18
現役マ○ドナルド店員まりあちゃんの援交!
無料で動画が見れます。
http://www.geisyagirl.com/
56GET! DVD:03/08/03 18:21
☆★ 新商品 ゾク・ゾク 入荷!! 急げ〜!! ☆★☆
★☆★☆★☆★☆★☆★☆★☆★☆★☆★☆★☆★☆★☆★
☆★ 送料激安!  スピード発送!  商品豊富!   
★☆      http://www.get-dvd.com        
☆★ 激安DVDショップ 「GETDVDドットコム」 
★☆      http://www.get-dvd.com        
☆★ 今すぐアクセス Let’s Go!   急げ! 
★☆★☆★☆★☆★☆★☆★☆★☆★☆★☆★☆★☆★☆★
微分積分の本を読んでいたら
h→0 のとき tan h / h → 1
ってのがいきなり出てきました。どうやって証明するんでしょうか?
(sin h)/h→1
トップエリート街道さん、ありがとうございます。

AOsinα=OC
(AOsinα+W)tanβ=L
ということまではわかってるのですが、
AOをL,Wで表す方法がわからんのです・・・・・・

(3)ですが、前進ですかね?
6057:03/08/03 18:46
>>58
レスありがとうございます。解決しました。
61トップエリート街道さん ◆BIG6e4aEMg :03/08/03 18:57
>>59
先に三角形OBDを考えると、Rsinβ=L・・・(*)
OD=Rcosβだから、これを使うと、OC=Rcosβ-W
で、三角形OACを見て、(Rcosβ-W)tanα=L・・・(**)
(*),(**)からRを消去して下さい。
(3)は、車庫の入り口に(道路に垂直に)車がさしかかった絵を描いてみて、
回転の中心がどこに来るか考えて下さい。
62_:03/08/03 19:09
63_:03/08/03 19:15
6437:03/08/03 19:47
>トップエリート街道さん
おかげさまで(1)、(2)と解決できました。
本当にありがとうございます。

軌跡を書いたのですが、(3)は前進・後退ともに同じであっているでしょうか?
お願いいたします。
65トップエリート街道さん ◆BIG6e4aEMg :03/08/03 20:03
>>64
簡単の為に、車が車輪から前後にはみ出してないとすると、
後退で入る時は、回転の中心が道路の端に来るから、道幅はRあればいい。
前進で入る時は、回転の中心は道路の端からL離れた所に来るでしょ?
そうすると道幅はさっきの記号でL+OD=BD+OD>OB=R 必要になる。
66vcc:03/08/03 20:21
★★★★★★★★★★★★★★★★★★★★★★★★★★★★★★★★★
<コピペ推奨>「ゆうくんストリート」
http://www.geocities.co.jp/HeartLand-Oak/1314/
まずは↑のページを見て貰いたい。ビルダーで作成しているようなのだが、
入った瞬間に様々な画像が次々に動き回り、非常に見難い。というより、
何が何だか分からない状態である。また「最近メール来ないな〜。女の子の
メール待ってま〜す」「ビルダー使ってるけど僕はタグも分かる」など、イタイ発言も
満載。管理人はゆうくん(13・リア厨2)。「中学生ちゃんねる」以来のこの極悪
ホームページを殲滅させるべく、2ちゃんねらーの者共よ、集え!
<信用できない方のために、掲示板のアドレス>
http://www.ziyu.net/~rent/bbs/pt.cgi?room=yuukun
★★★★★★★★★★★★★★★★★★★★★★★★★★★★★★★★★
>>22
積分区間は[0,無限大]です。

トップエリート街道さん助かりました。
ありがとうございました。
68132人目の素数さん:03/08/03 23:18
x*log(x^2+1) を積分せよ。

誰か教えてくださいませ。
本当にお願いします。
>>68
y=x^2+1
で置換。
マルチウゼー
∫「0,2π」(1+cosinX^2)^(1/2)が解りません。教えてください。
>>71
問題は正確に写すこと。
>cosin

1文字だけ略したのか。
7471:03/08/03 23:46
∫[0,2π]√[1+{cosin(x)}^2]dx
これでいいですか?
75132人目の素数さん:03/08/03 23:49
γsin(θ+α) γ>0の形に変形して下さい
(1) sinθ+cosθ
(2)sinθ-√3cosθ
(3)2sinθ+cosθ

低脳な私では解けませんでした。
よろしくお願いします。
>>74
cosinってのは cosのことかい?
>>75
教科書や参考書に書いてある通り。
7871:03/08/03 23:55
>>76すみませんそのとおりです
>>75
例えば(1)なら
sinαとcosαが1:1の比で、
2乗の和が1になるように√2で割ってみると
√2((1/√2)sinθ+(1/√2)cosθ)
αはいくら?
8075:03/08/04 00:00
>>77
私は教科書すらあまり理解できないほどの低脳なんです。
お釈迦様が悪にくもの糸を垂らすように、私にも御慈悲をお願いします。
>>80
ここで教科書より丁寧な説明をしろと?
せめてどこが分からないかぐらい書けよ。
8275:03/08/04 00:05
>81すみません。
この問題でなぜαが出てくるのかが謎なんです。
全体的によく分かりません。
とか言わないように
>>82
どういう意味?
85132人目の素数さん:03/08/04 00:11
>>75
記号はαである必要ないんだよ。
αに何か数値を当てはめて同じような形にしろという問題なんだから。
8675:03/08/04 00:12
テキスト読んでるうちに少しずつ分かってきました。
もう少し自分の力で頑張ってみます。
8775:03/08/04 00:14
>>85さん
それを、気付きました。
もう少しやって見ます。
また分からなくなったらよろしくお願いします
>>74
とりあえずt=cosxとでもしてみれば?
89132人目の素数さん:03/08/04 01:11
522 :センスでカバー ◆skKtv5y4yk :03/08/03 22:17 ID:lG6837cr
>>論理というのは○か×かということだろ
>違う。
違わない。
論理というのは、○か×のことだ。
○か×以外を認めてしまっては、それはもはや論理とは呼ばない。
1+1の答えに2以外を認めてしまっては、数学は成り立たない

成り立たないんですかね?
90132人目の素数さん:03/08/04 01:13
>>89
定義次第でどうにでもなる。それが数学。

敢えて言うなら、Aであることと、Aでないことを
同時に認めると、全ての命題が真となる。

あと、「古典論理」では、全ての命題は○か×
(真か偽)に定まるものと決めて議論を始める。
               ~~~~~~~
非古典論理では、そうでないものもうじゃうじゃ出てくる。
92132人目の素数さん:03/08/04 01:21
>>91
そうですよね。数学ってもっと自由なものですよね。
数学の概念は自由とかそういうのとは独立した物でないかと。
自由よりもっと自由なわけだ
95132人目の素数さん:03/08/04 12:11
132人目の素数っていくつよ?
96_:03/08/04 12:21
98132人目の素数さん:03/08/04 12:39
2次関数ですが

不等式 2|x-1|+|x+4|<11 を解け

これが解かりません。解説を見ても意味が解らないので解ける方どうぞ教えてください。
絶対値記号間違ってたらスマソ
>>98
y=2|x-1|+|x+4|
とおいてグラフを描き、y=11の直線を引いてみる。
100132人目の素数さん:03/08/04 12:52
>>99
早いアドバイスありがとうございます。ですがy=2|x-1|+|x+4|のグラフが書けません。
これって展開するんじゃないですか?それともxに0.±1.±2...と代入するんですか?
>>98
|x-1|が正、|x+4|が正
|x-1|が正、|x+4|が負
|x-1|が負、|x+4|が正
|x-1|が負、|x+4|が負

の4通りに場合が考えられるが、
2番目は起こり得ないので、残りの
3つで分けする。
10298:03/08/04 12:58
>>100
場合分けするといいよ。解説ってやつも場合分けしてるはず。
絶対値が入った関数のグラフの描き方は教科書や参考書に
載ってる。
10398:03/08/04 13:00
|x-1|が負とか|x+4|が負 はありえないよ。
104132人目の素数さん:03/08/04 13:06
>>101>>102
絶対値が入った関数は絶対場合分けをするんですか?場合分けする対象は絶対値の中身でいいんですよね?
はい
106132人目の素数さん:03/08/04 13:07
改行しようとしたら書き込んでしまった…

>>103
何で有り得ないんですか?
107101:03/08/04 13:08
すまん、ミスった。
101は絶対値記号をアボーソしてくれ。
絶対値がついてるから
109132人目の素数さん:03/08/04 13:15
y=2|x-1|+|x+4|のグラフを描いてy=11の線引いて場合分けをして…その後はどうすればいいんですか?
何回もすいません(__)
y=2|x-1|+|x+4|のグラフがy=11のグラフの下にあるような
xの範囲を求めればいい。
答えがあるなら、自分が描いたグラフと解答のxの範囲を
比べてみると不等式とグラフの関係がわかるかも。
112132人目の素数さん:03/08/04 13:40
>>110>>111
ありがとうございました。あとは教科書見ながらグラフ描いてみます。
因みに解答では

2|x-1|+|x+4|<11...(1)

y=2|x-1|+|x+4|...(2)
y=11...(3)

とすると(1)の解は(2)のグラフが(3)のグラフより下方にあるようなxの範囲である。
(2)について
x<-4 のとき
   y=-2(x-1)-(x+4)
    =-3x-2
-4≦x<1 のとき
   y=-2(x-1)+(x+4)
    =-x+6
1≦x のとき
   y=2(x-1)+(x+4) 
    =3x+2

<ここで(2)のグラフを描く>
(2)(3)の交点のx座標は
x<−4のとき
-3x-2=11よりx=-13/3
1≦xのとき
    3x+2=11よりx=3
したがって図より(1)の解は -13/3<x<3…(答)

これってレベル的には簡単ですか?
113132人目の素数さん:03/08/04 15:03
easy
114132人目の素数さん:03/08/04 15:14
あるサイトを見て疑問に思ったので頼む

>”1+2=3”は’3’の定義であることもあります。
>”2+1=3”は定理
と書いてあったのだが定義と定理が逆ではないかどうか
判断をお願いします
115132人目の素数さん:03/08/04 16:13
>114
どういう定義を採用しているかによります。
その部分からだけではなんともいえません。
加法の定義の基礎にある、1を加える(次の自然数を作る)
という操作をどちら向きにとるかによる。
右から足す方が感覚的には自然だろうが、結果的に構成できるものは同じ。
これだけ時間空いててもかぶる事もあるんだな
118132人目の素数さん:03/08/04 16:22
2次関数の問題です。

グラフが次の条件を満たす2次関数の式を求めよ。
(1)軸がx=-2で点(-3,4)を通り、x軸と接する。
(2)2点(2,-5)、(-1,-2)を通り、y切片が3。
(3)x軸と2点(-1,0)、(3,0)で交わり、点(1,-8)を通る。
(4)3点(1,0)、(-1,8)、(4,3)を通る。

初歩かと思いますが、予習に当たるためわかりません。
どうかよろしくお願いします。
>>118
(1)x=-2を軸とし、x軸と接するという条件のもと、
平方完成された状態の式を書いてみる。
頭の係数は点(-3,4)を通ることから決定する。

(3)同様にx軸と2点(-1,0)、(3,0)で交わる2次関数を
因数分解された状態で表し、残り1点を通ることから
係数を確定。

(2)(4)与えられた3点を通るわけだから、
それぞれの条件式を3つ連立させれば各係数が決まる。
120プリンスーメロン:03/08/04 17:01
数式を立てるとき+−×÷を使いますが、これらの記号の意味って何なんでしょうか??
+ 荒々しい突きを巧みな剣さばきでかわし
− 時には空振りもする
× 激しいつばぜり合いは実力伯仲
÷ しかしついにまっぷたつに
122132人目の素数さん:03/08/04 17:14
☆★最新アダルト情報★yahoo検索エンジンからのプレゼント!!☆★
http://endou.kir.jp/marimo/link.html
123118:03/08/04 17:15
>>119さん
ありがとうございます。判りやすく、大変助かりました。
また、自分で考えてもわからないときにお力をお貸しいただけると
嬉しいです(笑)
誠にありがとうございました。
125132人目の素数さん:03/08/04 20:05
ちょっと確認してもらえませんか

1+1=2の証明

1+1≠2であると仮定する
右辺を左辺に移項する
1+1ー2≠0…(1)
(1)式は1+1≠2であることに矛盾しているゆえに
1+1=2である。
>>125
2 の定義おしえてもらえますか?
>>125
>1+1ー2≠0…(1)
>(1)式は1+1≠2であることに矛盾しているゆえに
これは何で?
あと+の定義
- の定義もね
(1)式と1+1≠2がどう矛盾するんだろ
>>125
移項って何?
>>125
ついでに、
2>1も証明して下さい。
133ちょっと確認してもらえませんか:03/08/05 00:03
皆さんならどう証明していきますか?
135ちょっと確認してもらえませんか(マルチ):03/08/05 01:03
>>134
答えがいまいちだね。。。
定義だのこういうものだのって
まったく参考になりません
へぇ
>>135
とりあえず>>130さんの疑問に答えるべし。
138132人目の素数さん:03/08/05 01:38
1+1ー2≠0…(1)あぁ〜これね
左辺2−2≠0計算して・・・
数学苦手で・・・わりぃな。これでも精一杯やってんだ
>1+1ー2≠0…(1)あぁ〜これね
>左辺2−2≠0計算して・・・
1行目から2行目にいくとき1+1=2つかってるような・・・
1+1=2 を仮定したら、1+1=2が出てくるのは当然の結果だわな。
141140:03/08/05 02:27
結局
みんなわからないわけね(稿)
142132人目の素数さん:03/08/05 02:51
質問させてください。

2^n+3^n < 10^10  (n≧1)をみたす最大の正の整数nを求める、という文脈で、
2^n+3^n < 10^10 ≦ 2^(n+1)+3^(n+1) ・・・(A)
を利用して題意のnを求めたいのですが、解答には、(A)のためには
3^n < 10^10 < 2*3^(n+1)  ・・・(B)
が成り立つことが必要だとあります。そして(B)の常用対数を考えてnを求めるのですが、
(A)から(B)への論理的なつながりがわかりません。どのように考えればよいのでしょうか。
2^(n+1) < 3^(n+1)
>>142
3^n<2^n+3^n、2^(n+1)+3^(n+1)<2×3^(n+1)なので(A)⇒(B)。
(A)のままだと計算もままならないので条件としてはよわいが計算しやすい(B)を利用するという感じ。
145140:03/08/05 02:59
>>141騙らないでください
146132人目の素数さん:03/08/05 03:17
高校の宿題なのですが、
僕は数学が苦手なのでイマイチわかりません。
レベルが低い質問で恐縮なのですが、
よろしかったらお答えいただけないでしょうか?

a^x+a^(-x)=tとおくとき、tの最小値を求めよ。またそのときのxの値を求めよ。
>>>146 左辺に相加相乗平均の不等式を使うとどうなる?
相加相乗平均ですか。
つまりa^x=kとおくと、
t=k+1/k≧2*√(k*1/k)=2
ってことですよね。
それでa^x=k>0ですから
t≧2ってことでしょうか?
あー、成る程、相加相乗平均をこういう時は使うんですね。
分かりました、ありがとうございました!
正確にはそれは正しくない
150132人目の素数さん:03/08/05 04:23
>>144
142です。ご説明ありがとうございます。理解できました。

余談なんですが、本問に限らずこうした言い換えをするときは、
おっしゃるように、条件の強さが変わることがありますが、そう
した変化が最終的な解答に影響を及ぼすこともありえますよね。
本文の場合は、(A)から(B)へと条件をゆるくした上でも答え
の候補が一つ(n=20)にしぼられるのでよいのですが、もし
類題で、複数候補が生じた場合、しかもそれら候補を(A)に相当
する条件に代入して検討するのが困難な場合はどういう対応を
とればよいでしょうか。しつこくてすみませんが。
151@ちゃん:03/08/05 05:28
『…万…億…兆…』の続きをすべて教えてください。
…万…億…兆…神武…綏靖…安寧…懿徳…孝昭…孝安…孝霊…孝元…開化…崇神…
153132人目の素数さん:03/08/05 05:41
152 ♯(-"-メ)
>>152
ワロタ
>>151
聞く前に検索しろ馬鹿
歴代天皇?
>>150
問題に依る。
マニュアル化にはしるのはやめた方がいい
超幾何関数はなんであんなカッコイイ名前をしているのか教えて下さい。
行列のべき乗exp(A)には一意性があるのでしょうか?(expA=expB→A=B?)
160 ◆BhMath2chk :03/08/05 10:00
i^2=−1。
exp(2πi)=exp(0)。
>>150
それは、その(A)⇒(B) と弱める作戦が失敗だったということだ。
もっとうまくいくような(A)⇒(C)を探すことになる。
162132人目の素数さん:03/08/05 11:49
>159
幾何級数というのは等比級数のことです。
その一般化なので超がつきます。
式を見ると一意性ではなく、単射性のように思われますが?
自分で考えてみてください。
163132人目の素数さん:03/08/05 12:28
ζ(s)がs>1で収束する証明を教えてください。検索しても「簡単に証明できる」とか「解析概論を参照」とか出てくる
ばっかりで、証明そのものが見つからないのですグスン。
>>163
∫(1/x^s)dx で押さえ込めばいいのではないか。
165132人目の素数さん:03/08/05 13:02
>>164さま
それだけでは全然わかりませんグスン。わかる人には自明なのでしょうがそこをなんとかお願いしまそ。
166132人目の素数さん:03/08/05 13:13
>>165
まず、ζ(s)の定義を書いてみてください
>>165
グラフ上に (1,1) (2,1/2^s) (3,1/3^s) (4,1/4^s) …
と点をプロットしていき、階段のような棒グラフを描く。
この階段全体がζ(s)の値になる。

一方、1/x^s は上記の全ての点を通るので、
階段棒グラフ<∫(1/x^s)dx
168163=165:03/08/05 13:43
>>166
「ゼータ関数」でググる。

>>167さま
もしかして、s≠1のとき、[1,∞]∫(1/x^s)dx = -1/(1-s)ってことで終了?
s=1のときは∫(1/x)dx=logx+cだから発散する・・・って訳か。
お騒がせしました。どうもありがとう!
ただ煽ったりする人がいてバカバカしいと思っても面倒で答えないと
いう状況もあるようだから、今後のために書いておこう。
1+1 というのは環において乗法単位元 1 について加法を使った結果
という意味もあるが、大抵は自然数としての和のことをいっている
場合である。自然数は次のように定義される。(1 から始める流儀)

(1) 1 は自然数である。 (2) n が自然数のとき、n' は自然数である。
(3) (1),(2) によって定義されたもののみが自然数である。

次の足し算の定義。
(1) n が自然数のとき、n + 1 = n' である。
(2) n,m が自然数のとき、n + m' = (n + m)' である。
(足し算が定義されることは数学的帰納法で証明される。)

2 は 1'、3 は 1''、4 は 1''' 、、、と定義される。この流儀だと
1 + 1 = 2、2 + 1 = 3, は 2,3 の定義と足し算の定義から明らか。
しかし、1 + 2 = 3 は 1 + 1' = (1 + 1)' = 2' = 3 という手順で
証明されることとなる。
定義は後付け、ややこしく言うだけ無駄

> (1) 1 は自然数である。 (2) n が自然数のとき、n' は自然数である。
> (3) (1),(2) によって定義されたもののみが自然数である。

不正確
>>171
< N, ' , 1,> of type <1,0> が自然数であるとは、以下を満たすことである。

(1) ∀x (x'≠1)
(2) ∀x,y(x'=y'⇒x=y)
(3) Nの部分集合Mが以下を満たすならばM=N
  (i) 1∈M
  (ii) x∈M ならば x'∈M

これでどうよ
173132人目の素数さん:03/08/05 14:53
3人の囚人A、B、Cの内、2人までが処刑され、
1人は釈放されることになっている。

Aは看守に尋ねた。
「B、Cの内、少なくとも1人は処刑されるわけだから、
とりあえず一人処刑される人を教えてくれないか?」

すると看守はこう答えた。
「Bは処刑されるよ。」

Aは少しホッとした。
自分が処刑される確率が2/3≒66.6%から1/2=50%に
減ったと思ったからだ。

看守はウソをつかないものとして、
本当にAが処刑される確率は減ったのだろうか?
>>171
この定義は自然数論のなかで、ゲーデルの不完全性定理を証明
する際、コード化したものにも通用する定義で、>>172 のもの
だと集合の概念がないと定義にならない。
もし不正確というなら、なにが正確なものか書いてみたら?
わからなければ専門家にきいてきたらわかると思いますが。
不正確かは知らんが不十分だな
ということはやはり不正確か
>>175
不十分って、なにが不十分なの?
169 のこれって数理論理学の本に書いてあるとおりだよ。
集合 {0,1} に対し、1'=0、0'=1と決める。
するとこれは定義を満たすので、
{0,1}が自然数の全体ということになってしまう。
>>176
ペアノの公理を見れば明らかだろ
少なくとも他に循環しないことを含める必要がある
180132人目の素数さん:03/08/05 15:42
>>169
> ただ煽ったりする人がいてバカバカしいと思っても面倒で答えないと
> いう状況もあるようだから、今後のために書いておこう。

今後のために、あなたの言わんとしていることを詳しく書いて下さい。
以下の3問、お願い致します。

・2次関数y=x^2-2ax+2 (0≦x≦2)の最大値と最小値を次の場合について求めよ。
(1)1<a<2のとき
(2)a≧2のとき
・x=-1のとき最大値8をとり、x=-3のときy=5となる2次関数を求めよ。

考えたのですが、どうしても答えが合いません。
どうかご助力お願いします。
>>180
178 のような解釈ではない。n という対象(文字)があったとき、この
文字の左上に ' というものを付けた n' という対象を自然数とする、
というのが (2) の意味するところ。(3) はこの定義が数学的帰納法に
よる定義であるという宣言。一般に帰納的定義は、論理式で書かれて
いない。論理式で書く(Explicitly definable という)には 172 の
形式で集合についての量化子を使うものもあるが、これだと数論での
論理式とはならない、そこで有限列のコードを使って書くのが普通。
しかし、どちらも帰納法が許される理論ですることで、もとの概念と
して 169 にあるものが普通であるということ。
言わんとしていることは、1+1 =2 は定義だといっても、定義がどこ
にも見つからないと具合が悪かろうと思って書いただけ。
おかしいと思う人はよく調べられたらよいでしょう。
183182:03/08/05 16:07
>>182 訂正: 左上でなく右上
ん?
>169の文章だけで、>178の解釈はおかしい。
ということが出てくるのか?
185132人目の素数さん:03/08/05 16:22
>>181
y=x^2-2ax+2
=(x-a)^2-a^2+2
1<a<2のときは、x=0のとき最大、x=aのとき最小をとる。
よって最大値2(x=0),最小値-a^2+2(x=a)

a≧2のときは、x=0のとき最大、x=0のとき最小をとる。
よって最大値2(x=0),最小値-4a+6(x=2)

求める関数はx=-1のとき最大値8をとる二次関数なので、
y=a(x+1)^2+8 (a<0)とおける。
この関数が(-3,5)を通るので、代入すると
5=a(-3+1)^2+8
a=-3/4
よって求める関数はy=-3/4(x+1)^2+8
653 名前:132人目の素数さん 投稿日:03/08/05 15:40
1たす1のこたえはなんですか

しんしょうしゃがっこう いちねんいちくみ ふるかわなつみ


654 名前:aaad ◆ozOtJW9BFA 投稿日:03/08/05 16:09
なつみさん懐かしいなー。三年のとき一緒だったっけ。何で身障者学校にいるんだ?
普通だったと思うんだけど。どうしたのかな?
まあいいや。

答えは、2です。
1を足すということは、次の数を調べるということで、1の次の数は2です。
↑まじれすしてよかったかな?
>>178に付け加えて0=1と決める。
すると{0,1}={0}。
>>184
文字はいろいろな解釈が可能です。自然数を定義するときの普通の解釈
を前堤にするということです。
169 の定義に問題を感じている人で172 のように別案を出すことのできる
ひとは自然数の定義を自分なりにしている人ですが、大抵の人は自分で
自然数の定義をしていないで、何となく成立することを当然として受け
止めていることが多いのでしょう。
数学の研究でそれが問題となることは普通ないわけですから。
189バカ美:03/08/05 16:35
+−×÷とは何か
本源的な証明であるかどうかはさておき
@自然数Nの場合は
+と×に関する結合律しか満たさないので半群と呼ばれる。
−や÷はまだ登場しない。
A整数Zの場合は
加法について逆元が存在する。これをaに対して(−a)とする。
ここで−が登場した。
ちなみにZにおいては3つの演算(×+−)が登場し、
環の条件を満たす。
C有理数体Qの場合は
Qは可換体であり、全てのQの元は先ほど述べた
Qの部分環である整数環Zの元(整数のこと)
a,bによってa×b^-1と表され、ここでa÷bが登場。
ここで+×−÷全て登場した。
ちなみに四則の登場の過程では、半群⊂環・加法について群⊂体・乗法についても群
という具合に、登場に必要な定義が拡大していったが、
置換群、A+B=A∪B−A∩B A・B=A∩Bの演算のように、
群にはなるが環を定義できないもの、
環にはなるが体を定義できないものなどがある。(他にもいろいろな
演算がある。)
つまり、四則は数ある演算の中の特殊な例である。
これでいいかしら?



なんだ電波かよ。
>>189
いいんじゃない? (置換群のとき位数が、1、2のとき
集合でも1元集合のときは例外だね、大した問題じゃないけど。)
192132人目の素数さん:03/08/05 16:46
自分なりの定義って・・・
素数を2,3,5・・・と次々に出していく方程式を発見したら、
リーマン予想も解けちゃいますか?
>>192
数学の世界で通常おこなわれている定義のなかで、どれを採用するか?
ということ。どの定義も解釈できる場で、一致しないような定義では
数学ではこまる。しかし、たとえば、集合が有限であるという定義は
論理的には同値でないが、それなりに自然なものがいくつかある。だ
から、定義が論理的に唯一であると思うのは正しいとはいえない。
しかし、数学では定義しなくては始まらないから、最初に自分なりの
定義を書くということ。
195181:03/08/05 17:04
>>185さん、ありがとうございます。
丁寧な説明、とても参考になりました。
また機会がありましたらお願いします。
証明と自明の境界線を教えて欲しいんですけど、
そういうものって存在するんですか?
人によって全然違う。
証明が自明なんだよ
>>196
昔、A立先生という偉い先生は
とある大学で講義をしていたときに
ある定理の証明をお忘れになったようで
しばらくあーでもないこーでもないと考えた後
自分の書いた本に証明が載ってる筈だと
自分の研究室に戻られました。

先生は、教室に戻ってくるなり
「これは自明でした」
と言われ、何事もなかったように講義を続けられました。
200理解力ゼロ過ぎ:03/08/05 20:57
m, n が有理数で m√2 + n = 0 ならば m = n = 0 の証明が理解できません。
(証明)m != 0 とすれば √2 = -m / n となり、矛盾する。(中断)
↑ここがわかりません。これは m√2 + n = 0 => m = 0 を証明しただけじゃないですか?
(再開)ゆえに m = 0 。m = 0 ならば、これをもとの式に代入すると n = 0。(終わり)
と教科書はなってますす。
m != 0
はmの階乗が0なのかm≠0なのか。

まぁm≠0なんだろうけど、いちいちm !=0って書くのが気に食わない
m\=0
m<>0
≠です。紛らわしくてごめんなさい。いちいちっていうよりこっちの方が
書きやすかったからつい。
204132人目の素数さん:03/08/05 21:19
>>1がまた立てましたよ。
このネタしか知らないのですかねぇ
\__  _______
     |/        ,,,,,,, _
             /''''  '';::.
  /二⌒"''ヽ    l ≡   );;;:   / ̄ ̄ ̄ ̄ ̄ ̄ ̄
  〈i   `'ヾ |    ≧〒≦  :;/)  | ツマラン!!
  |こi .iこ ヾl    iー/ i ー'  k.l <  おまいの話は
  l / !.ヽヽ i6.    l ノ‐ヘ   iJ   |   ほんとツマラン!!
.  l,〈+ヽ ノ     U乞 し ノ     \_______
   ヽー '/      `ー ‐
>>200
有理数の意味わかってる?
206200:03/08/05 21:33
2行目の途中
√2 = -n / m でした。
>205
なんとなくしかわかりません。
>>200and>>206
オレは分かる、お前は分からない、それで良いじゃないか
>>206
問題の1行目って「m,nを整数とする」の間違いじゃない?
有理数っていうのは整数m,nを用いてm/nと表される数のこと。
ルート2はそう表されないでしょ?
だから2行目は矛盾してるんだよ
ん?有理数でいいんじゃないの?
√2=-「n/m」の間違いだな。

n,mが有理数のときにこれが有理数になることは
既知としてるんだろう。
上で既に訂正されてた。

しかし>>200がどういう疑問なのかよく分からん
ありがとうございます。
>207
分かりたいです。それなら自分で理解しろというお叱りはもっともです。
すいません。
>210
その通りです。
>211
自分でもわかりません。すいません、下に整理します。

「m ≠ 0 とすれば √2 = -(n/m) となり、矛盾する。」
上の文は、教科書に載っている
    m, nが整数で m√2 + n = 0 ならば m = n = 0
の証明の1行目です。上の文だけでは、まず
    m, nが整数で m√2 + n = 0 ならば m = 0
であることを証明しているように思えます。これは間違いでしょうか。
また、上の文からなぜ「ゆえにm = 0。」が導かれるのかわかりません。
「ある数aに対して(5/2)(a+1)の値を計算し、少数第一位を四捨五入すると4になった。
このようなaの範囲を求めよ」
解法を教えてください。お願いします。
>>212
「ゆえにm = 0」というのは、
仮定に依存しない命題「m=0」が示されたいう意味じゃないよ。

この段階で仮定を落として「m,n:整数、m√2 + n = 0=>m=0」という命題を得ることも
できるけど、ここでは仮定を残したまま、いま導いたm=0からさらに議論を進めている。

最終的に、「m,n:整数、m√2 + n = 0」の仮定のもとでm=0を、さらにn=0を導いたところで
「m,n:整数、m√2 + n = 0=>m=n=0」と結論付けている。
>>214
3.5≦(5/2)(a+1)<4.5 ⇔ 13≦a<17
2/5 ≦a< 4/5
再びお願いします。

(1)放物線y=2x^2-x+4と直線y=3x+kが接するようにkの値を求め、そのときの
接点の座標を求めよ。

(2)方程式-3x^2-ax+a-2<0の解がすべての数となるような定数kの値の範囲を
求めよ。

(3)2つの2次方程式x^2-2x+a=0,x^2+3x-2a=0の一方が異なる2つの解をもち、
他方が解を持たないような定数aの値の範囲を求めよ。

(4)不等式ax^2+15x+b>0の解が-1/3<x<2である。このとき、定数a,bの
値を求めよ。

全て解いてみようとしましたが、(2)と(4)途中まで行けた程度で、
他の問題に至っては全くわかりません。
どうか、ご助力お願いします。
>>217
どの問題も共通して
判別式 b^2-4ac を使う
詳しくは教科書参照
219 ◆6BFHB7Ku.g :03/08/06 00:43
>>217
ワークブック形式にしてみますた。[ ]を埋めれば完成。。

(1)
2次方程式:2x^2-x+4=3x+k が重解をもつので、判別式=0が成り立つ。
これにより,k=[ア]と求まる.またこのとき、この2次方程式の解(重解)は
x=1であるから,接点の座標は(1,3+[ア])になる.

(2)
f(x)=3x^2+ax-a+2 とおく.下に凸である放物線:y=f(x) がx軸と共有点を持たないとき,
すべての実数xに対して,f(x)>0 となるので,題意を満たすためには,
2次方程式:f(x)=0 が実数解を持たなければよい.
したがって,2次方程式:f(x)=0 の判別式をDとおくと,D<0 が成り立つ.
これにより,a<[ア],[イ]<a となる.

(3)
x^2-2x+a=0 の判別式をD1とし,x^2+3x-2a=0 の判別式をD2とすると,
(D1)/4=1-a,D2=9+8a である.
D1>0 かつ D2<0 を満たすaの範囲は,a<[ア] である.
D1<0 かつ D2>0 を満たすaの範囲は,a>[イ] である.
したがって,求めるaの範囲は,「a<[ア],[イ]<a」である.

(4)
a=0 のとき,x>-b/15 となるので,不適.したがって,以下,a≠0 として考える.
f(x)=ax^2+15x+b とおく.不等式:f(x)>0 の解が-1/3<x<2 となるので,
放物線:y=f(x) は上に凸でなければならない.したがって,a<0・・・(A) である.
また,2次方程式:f(x)=0 の解が x=-1/3,2 となることが必要なので,
解と係数の関係より,
(-1/3)+2=-15/a・・・(B)
(-1/3)*2=b/a・・・(C)
が成り立つ.(A),(B),(C)をすべて満足するa,bの値を求めれば,a=[ア],b=[イ]となる.
これはたしかに題意を満たすので十分である.
成長したねぇ
おじさん嬉しいよ
>>219
あんた偉い。
ガンガレ>>217
222217:03/08/06 01:05
>>218
確かに教科書に載ってはいるのですが・・・。予習に当たるため、理解力の乏しい
俺にはなかなか難しいのですよ;;
>>219
スゴイ!ありがとうございます〜。
こんな形にまでしてもらえるとは・・・。お手数をお掛けします(苦笑)
これならなんとか判りそうです。本当にありがとうございます。

できる限り自信で解決するよう努めますが、また何か合ったときには
よろしくお願いします。
誠にありがとうございました。
223200:03/08/06 01:15
>214
何かを勘違いしているようです。わかりそうです。
ありがとうございました。
次の問題お願いします。

f(x)=a^2x+a^-2x-2(a+a^-1)(a^x+a^-x)+2(a+a^-1)^2

とする時、f(x)の最小値を求めよ。また、その時のxの値を求めよ。
>>224
s=a^x+a^(−x)とすると2≦sでa^(2x)+a^(−2x)=s^2−2。
226224:03/08/06 01:52
>>225
ありがとうございます。
置き換えという考え方が思い浮かびませんでした…。
その後は簡単ですね。
本当にありがとうございました。
をっさんがもの凄い勢いで・・スレで質問をしたのですが、移動を勧められてこちらへ来ました。


やばいくらいに、数学がわからないので教えて下さい。

逆関数の微分で、dy/dx = 1/(dx/dy) ってなるのはなんでなんですか?
左辺と右辺が等式(!)なのはわかるのですが、何をやっているのか
意味がよく解りません。
そもそも、逆関数というのは、グラフだとy=xに対称にひっくりかえして
(左上と右下を入れ替えて)、xとyの文字を入れ替えたものという認識で
いいのでしょうか?
また、そうだとして、ちょっと思ったのですが、
傾きがたとえば2の直線が1/2になるのと
この dy/dx = 1/(dx/dy) という式は関係があるのでしょうか?
>>227
もちろんある。グラフをしばらく眺めてみるといい
>>227
>逆関数というのは、グラフだとy=xに対称にひっくりかえして
>(左上と右下を入れ替えて)、xとyの文字を入れ替えたものという認識で
>いいのでしょうか?

OK。

>傾きがたとえば2の直線が1/2になるのと
>この dy/dx = 1/(dx/dy) という式は関係があるのでしょうか?

大いに関係がある。
微分というのは、曲線のごく短い部分を、直線で近似することに他ならない。
その近似直線の傾きを、曲線のその点における微係数という。
そう考えれば、逆関数の微係数がもとの逆数になるのは当然だろう。
>>228
そうですか。ありがとうございます。
しばらくグラフを観ながら、あれこれと考えてみます。
231227=230:03/08/06 03:00
>>229
レスありがとうございます。
なんか、わかってきました。
逆関数っていうのは、つまり、ある点での接線の傾きは、
元の関数の相当する点での傾きの逆数なんですね。

今、y = tan^-1 x の微分とかが出てきていて
ちょっと混乱気味なのですが、微分をするのに
この形だとややこしいので、元のtan xの形に戻すため
x = tan y という表現をして、1/(dx/dy)とするということ
なんでしょうか?
232132人目の素数さん:03/08/06 03:11
>>231
229じゃないけど、その通り。
y = tan^-1 x のとき、x = tan y だから両辺をyで微分して
dx/dy=1/(cos y)^2
1+(tan y)^2=1/(cos y)^2 が成り立つから
dx/dy=1+(tan y)^2
x=tan y を代入して
dx/dy=1+x^2
ここで、逆関数の微分の公式を使うと、
dy/dx=1/(dx/dy)=1/(1+x^2)
このようにすれば、sin x や cos x の逆関数も計算できる。
>>231
ライプニッツをやめてニュートンの記号で書けば
掲示板の都合上簡便にf(x)の逆関数をg(x)とおいて
g'(x)=1/f'(g(x))
いやラグランジュの記法だったかも
\dot{y} (上ドット付きy) ニュートン
dy/dx ライプニッツ
f' ラグランジュ
f_x (f下付きx) オイラー クレロー
D_xf,Df ベルヌーイ?

調べてみたらこんな感じらしい
236132人目の素数さん:03/08/06 03:58
ある牧場では200頭の牛を飼うと180日で牧草を食べ尽くし、
180頭飼うと240日で食べ尽くすという。この牧場で160頭の牛
を飼った場合、牛は牧草を何日で食べ尽くすか?ただし、
牧草は毎日一定の速さで成長し、牛は毎日牧場全体の牧草を均等
に一定量だけ食べるものとする。

お願いしまつ・・・
237132人目の素数さん:03/08/06 04:06
>>236
微分方程式知っているの?
238_:03/08/06 04:06
>>326
それはニュートン算といって方程式で解くと何も考えずに済むし
ここに書くのも簡単だけど、昔ながらの工夫された解法がある問題で
出題者の意図もおそらくそこにあるでしょう。
だから「ニュートン算」で検索した方がいいと思う。
図付きでわかりやすい解説があるはず
240237:03/08/06 04:15
俺恥ずかしいな。すまない
241132人目の素数さん:03/08/06 05:04
>>236
360日かな?
120頭以下だと永久になくならなくてエコロジー。
243132人目の素数さん:03/08/06 05:35
高校数学ですみません・・
ある式を(x-2)で割ったときの余りが3、(x-3)^2で割った時の余りが0
この式を(x-2)(x-3)^2で割ったときの余りは? という問いなのですが、
余りを二次式ax^2+bx+cとおくと
f(2)=4a+2b+c=3
f(3)=9a+3b+c=0
となってしまい解けないのです・・お願いします・・・
ax^2+bx+cは(x-3)^2=x^2-6x+9で割り切れるのだから…
245132人目の素数さん:03/08/06 06:06
別の板で無視されたのでこちらでお尋ねします。

全分散の公式V(X)=E(V(X|Y))+V(E(X|Y))を示して頂けないでしょうか。
ヒントだけでもお願いします。
>>243
n,m,lは自然数。
f(x)=(x-2)n+3
=((x-3)^2)m
=(x-2)((x-3)^2)l+ax^2+bx+c
=(x-2)((x-3)^2)l+a((x-3)^2)+(b-6a)x+c-9a
=(x-2)((x-3)^2)l+(ax+2a+b)(x-2)+4a+2b+c

b-6a=0
c-9a=0
4a+2b+c=3

a=3/25, b=18/25, c=27/25

答、(3/25)x^2+(18/25)x+27/25
247132人目の素数さん:03/08/06 06:08
>>243
余りはa(x-3)^2と置ける
さて質問に答えてやった事だし寝るか。
249243:03/08/06 06:25
どうもありがとうございます>レスくれたみなさん

>ax^2+bx+cは(x-3)^2=x^2-6x+9で割り切れる
というのはわかっていたのですがこれをどう式にすればいいのかわからなくて・・
>246さんの計算法で追っていってみます。
250 ◆6BFHB7Ku.g :03/08/06 11:50
>>236
小学生かな・・。
ニュートン算を,方程式を使わないで解いたほうがいいのか,
それとも使って機械的処理に持ち込んだほうがいいのか迷うところだけど,僕は後者の方法ですた。。
頭が柔らかくするには使わない方がいいらしいけど・・。

>>241 あってると思います。。
>>242 ワロタ

ちなみに,121頭の牛を飼うと,食べ尽くすのに14400日(≒39.5年)かかります。。
120頭以下だと永久に牧草が食べ尽くされることはないのでエコロジーでつ。
エコロジーワラタ
252227=230:03/08/06 15:21
>>232
レスありがとうございます。
なんか、バッチリわかってきた気がします。

>>233
この表記だと、なんだか逆関数ともとの関数のxとyの関係がよく分かる感じがします。



それにしても、数学板って24時間いつでも人がいるんですね。ビックリしました。
>>252
この板は、2chの中では過疎板だと思う。。。
4つ以上の点を全て通る2次関数が存在するかどうか判定する方法って
ありますか?あるならそれを教えていただけないでしょうか?
どうかよろしくお願いします。
3点で2次関数決めて残りの点がその関数上にあるか
256数学野郎 ◆eNwncubcDk :03/08/06 21:57
因数分解で質問ですけど

http://rx.sakura.ne.jp/~yuhgi/high/img-box/img20030806215604.jpg

ですが、マイナスを消す操作はどのようにしたのかもし時間の空いてる先生
いましたらよろしくお願いします

           ;:'´
        _....._{{ 〃
      , - ' ,..、、.ヾ{{フ'⌒`ヽ、  
    /  ,:', -‐‐` ´ '´⌒ヽ ヾ:、
.   ,'   ,'´ ,ィ ,ィ ,' ,   `ヽ',  ',-
    ,'   .i  /|. /.| { i,  i,  }.  }_,,
   ! |  ! .,'-.{ ! !|; |`、.}゙!.! |.  ! 
   ', ', |Vァ=、゙、 `゙、!-_:ト,リ', l ! |  
    ヽ、', l:!Kノ}.     f:_.)i゙i: リ ! l    _______
     | l!iヾ- ' ,   .!__:ノ ゙ ,リ l リ'  /私ここで待ってるね。
.     ',|!!、    r‐┐   ` ノ' /,イ  <http://www.geocities.co.jp/MusicStar-Bass/6678/nicebody.swf
      'i!゙、ヽ、 ゙ー'  _, ィ,:',:''´    \_______
      . >  ⌒ヽ
       /    へ \
      /    /   \\
      レ  ノ     ヽ_つ
     /  /
    /  /|
   ( ( 、
    |  |、 \
  .  | / \ ⌒l
    | |   ) /
   ノ  )   し'
  (_/
>>256
 -(a^2-b^2)c+(a^2-b^2)b
=(a^2-b^2)b-(a^2-b^2)c
=(a^2-b^2)(b-c)
です。
259132人目の素数さん:03/08/06 22:04
Nが3以上のとき
N(N-1)a_[n+1]=(N-1)(N-2)a_[n]のA_[N]を求めよ
解。(N-1)(N-2)a_[n]=2*1a[3] 故にa[n]=2a[3]/(N-1)(N-2)
何でこうなるんでしょうか?
>>256
マイナスって上の式の最初についてるやつだよね
それは(b-c)のマイナスになってる
261数学野郎 ◆eNwncubcDk :03/08/06 22:12
>>258
先生ありがとうございます。理解できました。

>>260
ありがとうございました。理解できました。
>>256
なんでわざわざ写真?
>>259
n(n-1)a_[n+1]
=(n-1)(n-2)a_[n]
=(n-2)(n-3)a_[n-1]
・・・
=2*1*a_[3]
よって
(n-1)(n-2)a_[n]=2*1*a_[3]

今回はさくらスレのテンプレがアレだったから見逃すけど
今度マルチやったら誰も答えないよ
>>259
a_(n+1)/a_n = {(n-1)(n-2)}/{n(n-1)} = (n-2)/n
a_n/a_3 = {a_n/a_(n-1)}{a_(n-1)/a_(n-2)}…(a_4/a_3) = {(n-3)/(n-1)}{(n-4)/(n-2)}…(1/3) = (3*2)/{(n-1)(n-2)}
∴ a_n = 2*a_3/{(n-1)(n-2)}
265132人目の素数さん:03/08/06 22:37
1 + (-x + x^3/3! - x^5/5! + ・・・) + (-x + x^3/3! - x^5/5! + ・・・)^2
+ (-x + x^3/3! - x^5/5! + ・・・)^3 + (-x + x^3/3! - x^5/5! + ・・・)^4 + ・・・

を4次か5次の項まで書き下したいのですが、どう計算すればいいですか?

1 - x + x^2 + x^3/3! - x^3

くらいまではわかるのですが
>>265
それぞれの項の最低次数を求める。
>>265
1 + {-x + x^3/3! - x^5/5! + o(x^6)} + {-x + x^3/3! - x^5/5! + o(x^6)}^2
+ {-x + x^3/3! + o(x^4)}^3 + {-x + x^3/3! + o(x^4)}^4 + {-x + o(x^2)}^5 + o(x^5)
を展開したら?
268132人目の素数さん:03/08/06 23:27
>>266
最低次数の項のみではできません。

>>267のように低い次数のをうまくとって展開したらできました。
計算が面倒なだけでした。
269ぼるじょあ ◆yEbBEcuFOU :03/08/06 23:37
>268
>最低次数の項のみではできません。

最低次数を求めて、除外できるところを確定するという意味だと思うんだが?
270132人目の素数さん:03/08/06 23:46
それならわかりますが、
"のみでは" とも書いています。

n次まで求めたいときは、
(-x + x^3/3! - x^5/5! + ・・・)^n
で(-x)^nまで求めて↑の多項式の他の項と、
(-x + x^3/3! - x^5/5! + ・・・)^k の n+1次以降を
求める必要がないことはわかります。

n-1次以降は左下から右上に線を引いたとき
線の上側は最低次でなくとも必要になります。
271132人目の素数さん:03/08/06 23:47
訂正:n-1次以降 → n-1次以前
272132人目の素数さん:03/08/06 23:47
ぼるじょあ ◆yEbBEcuFOU さんに質問です。
地理お国自慢住民でもあるのですか?
1/(1+sinh(x))
>>272
心と宗教板でも見かけた事が有るぞい
ぼるじょあは共有コテだからその質問は無意味では?
276132人目の素数さん:03/08/07 00:50
>>254
2次関数を y=ax^2+bx+c とする。これで表される2次曲線が、
平面上の4点(x1,y1)...(x4,y4)を通るものとすると
y1=ax1^2+bx1+c
....
y4=ax4^2+bx4+c の4つの式が成り立つ。
4×4行列Aを
A=
x1^2 x1 1 y1
.....
x4^2 x4 1 y4
とおくと上の4つの式は
A(a,b,c,-1)'=(0,0,0,0)' ('は転置を表す。)
と表される。A^(-1)が存在すると左からかけて(a,b,c,-1)'=(0,0,0,0)'
となり矛盾するので、A^(-1)は存在しない。よって、
異なる4つの点を通る2次曲線が存在する⇒ detA=0
同じように考えれば、4つ以上の異なる点を通る2次曲線が存在するなら
n×4行列Bを
B=
x1^2 x1 1 y1
.....
xn^2 xn 1 yn
とおいたとき rank B<4 となる。
277132人目の素数さん:03/08/07 01:36
積分なんですが
ln(sinx)dx
が解けません・・・どなたかご指導を・・・
278132人目の素数さん:03/08/07 01:47
今パチスロ板でいろんな板の人に説教されたい!というスレが建ってます。
よければお越しいただけないでしょうか。
厳しくお願い致します!名前欄にどの板から来たか書いていただけると幸いです。
   
http://gamble.2ch.net/test/read.cgi/slot/1060184797/l50
279132人目の素数さん:03/08/07 02:05
↑レス消費が激しいときのさくらスレ・くだスレ状態。
まともな奴がのけ者にされているスレ。行くだけ無駄。
-cos(x)
281132人目の素数さん:03/08/07 03:10
280さんどうもです。
どうやって解いたらいいんでしょうか
>>281
礼を言う前に検算ぐらいシロ!
 ・ ・
0.2x=1.01
のような循環小数の方程式って
循環小数を分数に戻して解く以外方法はありませんか?
って、すごいずれてる……
1.01(少数第二位の1の上に循環記号)です
割り算は上位からだからそのまま出来るだろ
286taylor:03/08/07 11:59
dy/dx + z = sin(2x)
dz/dx - y = 0
という問題があって、yとzはxの関数で、yとxの一般解
を求める問題があるんですけど、よく分かりません。

dy/dx + z = sin(2x) の両辺を微分して、
d^2y/dx^2 + dz/dx = 2cos(2x)
を、dz/dx - y = 0 に代入すると、

2cos(2x) - d^2y/dx^2 = y
つまり、y'' + y = 2cos(2x)
となるのですが、これは特性方程式も使えないし、何か
他に方法ってあるんですか?
よろしくお願いします。
そこまでいったらラプラス変換なりなんなりするしかないけどラプラス変換しても
仮にラプラス変換できてLy=(λの式)まできたとしてもこの(λの式)の逆変換を
おぼえてないとむずかしいとおもう。もちろんラプラス逆変換を直接計算することも
できるだろうけど。でもこの問題ならy''+y=cos2xとなるyはすぐ少なくとも一個はすぐみつかる。
実際形からしてy=kcos2xとあたりをつけて
y''+y=(4k-k)cos2x=cos2xよりk=1/3ととればよいことがわかる。
288132人目の素数さん:03/08/07 15:08
ヽ(`Д´)ノ   ボッキアゲスパイラル!
ヽ`Д´)
 (ヽ`Д)
 ( ヽ`)
(  ヽ
ヽ(   )ノ
ヽ   )
(ヽ  )
 (´ヽ )
(Д´ヽ
ヽ(`Д´)ノ
>>275
おいらもボルジョアになれる?共有コテだったとは知らなかった。
>>289
名前欄に「ぼるじょあ#セV8cLFセz」って書けばキミも今日からぼるじょあ◆yEbBEcuFOUだYO!
ヽ(゚∀゚)ノ ヨシ、サソークチョウセン


ヽ(゚∀゚≡゚∀゚)ノ イイカナ?イイカナ?


ヽ(゚∀゚)人(゚∀゚)ノ サイボー! ブンレツ !
293ねかま姫 ◆xdkteuOpHo :03/08/07 15:53
>>289
もっとかんたんに
ぼるじょあ#ぶるじょあ
でぼるじょあだYO!
>>293
似てるけど違うような
295ねかま姫 ◆xdkteuOpHo :03/08/07 15:59
どっかで見た記憶あったけどなぁー
スマソ
Σ(。д゚) キニ イラレテ シマッター
Linuxだと
ぼるじょあ#ぶるじょあ(#は半角)
でもオケでした。
ここで遊ぶな
299132人目の素数さん:03/08/07 16:07
馬鹿はこっちのスレに来るなよ。あっちへ帰れよ。
>>297
見る人のOSによって変わるのか
301GET! DVD:03/08/07 16:08
☆★ 新商品 ゾク・ゾク 入荷!! 急げ〜!! ☆★☆
★☆★☆★☆★☆★☆★☆★☆★☆★☆★☆★☆★☆★☆★
☆★ 送料激安!  スピード発送!  商品豊富!   
★☆      http://www.get-dvd.com        
☆★ 激安DVDショップ 「GETDVDドットコム」 
★☆      http://www.get-dvd.com        
☆★ 今すぐアクセス Let’s Go!   急げ! 
★☆★☆★☆★☆★☆★☆★☆★☆★☆★☆★☆★☆★☆★
302132人目の素数さん:03/08/07 22:31
すいません。
∫ln(sinx)dx
です。
おねがいします。
303132人目の素数さん:03/08/07 22:34
すいません。
∫sin(lnx)dx
です。
おねがいします。
>>302-303
マルチは市ね
y=lnxと置いて部分積分せよ
306\:03/08/07 23:29
なぜ四則演算+-×÷の四個の演算があって、六個とか八個
定義されていないのか、分かりません。だれか教えてください。
>>306
いきなりマジレスで申し訳ないが、
算盤で商業簿記をつけるにはそれだけあれば十分だからだ。
ですけどなんだ?
309math.1st ◆M9pCfogc9g :03/08/08 14:37
Re:>306 別に二項演算はいくらでも定義できるのだが。
四則演算は科学技術計算の基礎だから、特別目立っているだけだ。
310132人目の素数さん:03/08/08 17:33
質問。

Z_0:={L*(x^1)|Lは非負実数}∪{R*(x^0)|Rは非負実数}∪{C*(x^-1)|Cは非負実数}

Z_1:={x+y|x,y∈Z_0}∪{xy/(x+y)|x,y∈Z_0}⊃Z_0
.
.
.
Z_(n+1):={x+y|x,y∈Z_n}∪{xy/(x+y)|x,y∈Z_n}⊃Z_n
として、

Z:=∪Z_n (n∈N)とする。

Qの記号で全ての係数が非負の分数式を現す。
ZがQの部分集合である事は自明。

ZはQと一致するか?
一致しないとして、QはZに於いてちゅう密と考えて良いか?
311math.1st ◆M9pCfogc9g :03/08/08 17:40
Re:>310 Zには分母分子の次数の差が高々1になる分数式しかないような気がする。
あと、稠密ということは何かの位相が入っていると考えられるが、どんな位相で考えるのかを示して欲しい。
>>310
Z_0 の x は変数記号ですか? また、変数 は x だけでよいのでしょうか?
Z_(n+1) の定義で x,y はただ Z_n の要素をさすようですが、文字を変えた
方がよいでしょう。
x+y は 0 となりえますから除去すべきでしょう。

>>311 さんの答えのように、 1/x^2 は入りませんね。
無限にある自然数からでたらめに1個選んだときの
情報量は無限大のはずなのに、どの自然数も
有限長のビットパターンで表せるのはなぜですか?
314トップエリート街道さん ◆BIG6e4aEMg :03/08/08 18:29
有限長と言っても、長さはいくらでも長くできるからじゃない?
315132人目の素数さん:03/08/08 19:16
  ∧_∧ ッパシャ ッパシャ
  (   )】
  /  /┘
 ノ ̄ゝ  
ん?
317ビッグバン宇宙論は完全に大間違いだった!!!!!!!!:03/08/08 19:26
科学者よ、恥を知れ!
ビッグバン宇宙論は完全に間違いだった!
科学の原則を無視した、デタラメのインチキ理論だったのだ。
そして、そのビッグバン宇宙論の世界的な浸透は
アメリカ、ユダヤ・キリスト教勢力による世界支配のための思想的な戦略なのだ!
また、ビッグバン宇宙論の思想によって戦争が起こり、
貧富の差がひらき、終末的な絶望感が世界に蔓延しているのだ。
ビッグバン宇宙論は世界の平和を揺るがす、悪の元凶となっているのだ。
ビッグバン宇宙論とは、
「宇宙は『無』からビッグバン(大爆発)によって誕生した」という理論である。
この理論は、ユダヤ・キリスト教の創造神話(神が天地を創造した)そのものである。
ビッグバン宇宙論の実態は、科学理論ではなく宗教思想なのである。
『無』は科学的に証明できるものではなく、
そして、『無からの誕生』も科学では証明できるものではないのだ。
ビッグバン宇宙論が科学の正統であるという思想を、世界中の人々に
浸透させる戦略が成功したことにより、ユダヤ・キリスト教勢力の
世界における優位性が確立されていったのだ。(20世紀に)
そして、その思想的支配の最大の例が、アメリカやイギリスによるイラク戦争なのだ。
ビッグバン宇宙論の浸透により、世界中に終末思想(世界の終わり)が蔓延してしまっている。
そのことにより、自己中心的、せつな的、短絡的な考え方が社会に広がっている。
科学的に間違っているビッグバン宇宙論から脱却しなければならない。
そして、宇宙は無限だということを理解しなければならない。
人間は本当の宇宙観、世界観を構築し、
新しい時代に進んでいかなければならないのだ。
ビッグバン宇宙論が世界を支配している限り、平和な世界にはならないのだ。
そのことを科学者は重く受けとめるべきである。
平和の時代へ!!!!!!!!!!!!!!!!!!!!!!!!!!!!!!!!!!!!!
318132人目の素数さん:03/08/08 22:13
ヽ(`Д´)ノ   ボッキアゲスパイラル!
ヽ`Д´)
 (ヽ`Д)
 ( ヽ`)
(  ヽ
ヽ(   )ノ
ヽ   )
(ヽ  )
 (´ヽ )
(Д´ヽ
ヽ(`Д´)ノ
319132人目の素数さん:03/08/08 22:46
>>311>>312
ご指摘有難うございます。訂正しました。

変数Xの一変数分数式を考える。
Z_0:={L*(x^1)|Lは非負実数}∪{R*(x^0)|Rは正実数}∪{C*(x^-1)|Cは非負実数}

Z_1:={p+q|(p,q∈Z_0)∧(p+q≠0)}∪{pq/(p+q)|(p,q∈Z_0)∧(p+q≠0)}⊃Z_0
.
.
.
Z_(n+1):={p+q|(p,q∈Z_n)∧(p+q≠0)}∪{pq/(p+q)|(p,q∈Z_n)∧(p+q≠0)}⊃Z_n
として、

Z:=∪Z_n (n∈N)とする。

Qの記号で全ての係数が非負の分数式を現す。
ZがQの部分集合である事は自明。

ZはQと一致するか?
一致しないとして、
各点収束によって定義された位相でZの閉包を取るとQになるか?
一様収束ならばどうか?L2ノルムならばどうか?
>>313
自然数が帰納的に定義されてるから。
>無限にある自然数からでたらめに1個選んだときの

これ有名なパラドックスが生じるんだよな
何のパラドックスというんだっけ
322sage:03/08/09 00:04
x(1)=a (a>1) x(n+1)=1/3(2x(n)+a/x(n)^2) (n≧1)で
定められる数列x(n)について,x(n+1)-[3] √a<2/3(x(n)-[3] √a
であることを示し、lim_[n→∞]X(n)を求めよ。

よく解らないです・・・どなたか助けてください。
323132人目の素数さん:03/08/09 00:05
★☆ 夏休みは GETDVD で満喫・満喫!! ★☆★
★☆★☆★☆★☆★☆★☆★☆★☆★☆★☆★☆★☆★☆★
☆★ 送料激安!  スピード発送!  商品豊富!   
★☆      http://www.get-dvd.com        
☆★ 激安DVDショップ 「GETDVDドットコム」 
★☆      http://www.get-dvd.com        
☆★ 今すぐアクセス Let’s Go・Go!!   
★☆★☆★☆★☆★☆★☆★☆★☆★☆★☆★☆★☆★☆★
324132人目の素数さん:03/08/09 01:08
正六角形が円に内接しています。
このとき、六角形の周の長さと円周の長さは
比較するとどちらが長いでしょう?
質問です。
いま、合成関数の微分というのをやっているんですが、
対数微分法というのででてきた、
(d(ln y)/dy)*(dy/dx)というのがよく分かりません。
dy/dxは何度も見ているので分かるのですが、
(d(ln y)/dy)の意味がピンと来ません。
取り敢えず、分母のdのあとの文字で表された、分子のdのあとの式を
微分するというふうにとらえているのですが・・。
それと、対数微分法を適用しようとした方程式のyとln yの関係、
(d(ln y)/dy)の表すものがよく分かりません。どのように考えればよいのでしょうか?
???
>>325
>分母のdのあとの文字で表された、分子のdのあとの式を微分する

なんかちょっと変だな。
「分子のdのあとの式を、分母のdのあとの文字で微分する」でおk。

>(d(ln y)/dy)の表すものがよく分かりません。

だから、ln y をyで微分したものだから、1/yってことだ。
>>325
d(ln x)/dx なら分かるか?
今ちょっと思いついたのですが、
両辺を対数の式にしたものは、単に
y = ln u
u = f(x)
(表記が正しくないかもしれません・・)
の様な、合成関数なのでしょうか。
レスありがとうございます。

えと、分子のdのあとの式を、分母のdのあとの文字で微分する
ということは、分母のdのあとの文字の式になるんですね。

>>328
d(ln x)/dx = 1/x でしょうか。
>>325=>>330 ?
とりあえず、「関数」「導関数」の言葉の意味を復習汁。
あと、df(x)/dx が定義されることの意味もな。



332132人目の素数さん:03/08/09 05:13
1=0.9999…なんですよね?
だったら0.000…(無限ケタ)…0001=0となりますか?
理由も添えてお答え願います。
無限ケタと言っているのに最後の0001って何だよ?
0.1 0.01 0.001 0.0001 0.00001 …
という数列の極限という意味だろ。もちろん0だ
>>332は神!
無限ケタのその向こうまで見える
x^2の最小値は0。
無限個の値(1^2,(−π)^2,0^2)との比較ができる。
337132人目の素数さん:03/08/09 10:03
       人       
      (__)      
      (__)      
ウンコー  (・∀・,,)     
     O┬O )      
キコキコ ◎┴し'-◎ ≡
338趣味です。:03/08/09 11:22
女子高生が入ったあとのトイレにはすぐ入るようにしています。
そして排便したあとの残り香を鼻孔いっぱいに吸い込みます。
あと、たまにトイレの水槽に細工して水が出ないようにしときます。
待ち伏せして、女子高生が使用したあとみてみると5回に一回の
割合で「ブツ」が流れずに残ってます。
それをビニール袋にいれて持ち帰り、風呂場で体中に
塗りたくるのです。 至福の瞬間です。

通りすがりの初書き込みです.
>>332
記数法の問題です.>>334 さんの言うとおり無限小数は有限小数の小数点以下の
桁を増やした極限として解釈するというのが実数の10進の記数法です.
それで,小数点以下の桁を増やしていく数列に必ず実数の極限が存在するというのが
実数の連続性で, 実数が満たすべき公理として実数の定義に含まれます.
(正確なところは,高木貞治: 解析概論, 岩波書店 でも読んでください.ただし,
普通の高校生には勧めません.挫折します.)

実際,実数以外の数体系で 1≠0.99999.... を満たすようなものが作れます.
0.00...(無限桁)...01という0でない(実数でない)数をもつことも可能です. ( >>335 )
大小関係と四則計算くらいは定義できます.
参考に クヌース(D. E. Knuth): 超現実数, 海鳴社 をどうぞ. (この本の数体系の
オリジナルは J. H. Conwey: On Numbers and Games, Academic Pressです. )

1桁の加減算と乗算が1単位の手間で計算できるとします. 非常に大きい
n桁の数2個の乗算を最小の手間で行ったときの手間を F(n) 単位とします. 
F(n) はどの程度でしょうか.nに無関係な定数倍程度は無視します.

調べた限りでは n (log n) (loglog n) の(定数倍程度)の手間で
乗算ができます(Schoenhage, Strassen).
これより速いの知らないかということです.
既にある, 理論的にない, あるかも知れないが未解決, のどれでしょうか.
341132人目の素数さん:03/08/09 13:51
「x^2-x-(x^2-x-2)^(1/2)=4を解け。」
就職試験の問題集にあったんですけど、解き方教えてください。
(x^2-x-2)^(1/2)をyと置けば
与式はyの二次方程式
343132人目の素数さん:03/08/09 13:59
●乗法公式

[]にあてはまる数式を答えよ。

(2x + y)(4x^2 - 2xy + y^2)
=(2x + y){[ ア ]^2 - 2x * y + y^2}
=(2x)^3 + [ イ ]^3
=[ ウ ]


次の式を展開せよ。

@(2x - 1)^3
A(x + 2)(x^2 - 2x + 4)
B(x - 4)(x^2 + 4x + 16)
C(a - 1/3)(a^2 + 1/3a + 1/9)
D(2x + 3y)(4x^2 - 6xy + 9y^2)
344132人目の素数さん:03/08/09 13:59
●乗法公式

[]にあてはまる数式を答えよ。

x + y = Aとすると
x + y + z = [ ア ] + z
x + y + z = [ イ ] - z だから、
(x + y + z)(x + y - z)
=([ ウ ] + z([ エ ] - z)
=[ オ ]^2 - [ カ ]^2
Aをもどすと。
与式=([ キ ])^2 - [ ク ]^2
=[ ケ ]


次の式を展開せよ。

@(x + y + 3)(x + y + 5)
A(x + y + z)(x - y + z)
345132人目の素数さん:03/08/09 13:59
●乗法公式

[]にあてはまる数式を答えよ。

x + y = Aとすると
(x + y + z)^2
=([ ア ] + z)^2
=[ イ ]^2 + 2[ ウ ]z + z^2
Aをもどすと。
与式=([ エ ])^2 + 2([ オ ])z + z^2
=[ カ ]


次の式を展開せよ。

@(a + b + 3)^2
A(x + y + 2z)^2
346132人目の素数さん:03/08/09 14:13
342さん。ありがとう。341です。y=x^2-x-4として、その後どうするのでしょうか?
よろしくおねがいします。
347342:03/08/09 14:15
>>346
それで分からないヤツが就職するな
>>347
ハゲドー
349132人目の素数さん:03/08/09 14:39
整式 f(x)=ax^3+bx^2+cx+d において
f(-1),f(0),f(1)がいずれも3で割り切れないならば
方程式 f(x)=0 は整数解をもたないことを証明せよ

という問題なのですが解法を教えていただけないでしょうか
>>349
a,b,c,dは整数?

解法: 対偶を示せばよい。
ヒント: 整数 n に対して、 f(x+3n)≡f(x) (mod 3)
351132人目の素数さん:03/08/09 14:55
次の定積分を教えてください
 2
∫ √(2−x)dx
 0
t = √(2-x) とおいて置換積分
353132人目の素数さん:03/08/09 15:12
xの2次方程式 x^2-2kx-3k-9=0 が2つの整数解をもつように定数kを求めよ

という問題なのですが、教えてください
354132人目の素数さん:03/08/09 15:20
教えて下さい。
四角形を有限個の三角形に分割する事は可能ですが、
その逆(三角形を有限個の四角形に分割する)は不可能です。
これの証明はどのようにすればよいのでしょうか?
また、三角形を無限個の四角形で分割すると考えると、
有限の中に無限を閉じ込めることになって矛盾してしまいます。
数学的にはどのように考えるのが一般的なのでしょうか?
長くなりましたが、ご教示のほど宜しくおながいします。
>三角形を有限個の四角形に分割する
出来る。特に制限(直線で、など)がないなら
>>353
2つの整数解を持つならば、
それらの和 2k および 積 -3k-9 は整数なので、
k = 2(2k) + (-3k-9) + 9 も整数である。

縮約型判別式
D/4 = (-k)^2-(-3k-9) = k^2+3k+9
が完全平方数になるような k を求めればよい。

k = -1 のときは D/4 = 7 となり条件を満たさない。
k = 0 のときは D/4 = 9 となり条件を満たす。
k が正のときは
(k+1)^2 < k^2+3k+9 < (k+3)^2 なので k^2+3k+9 = (k+2)^2
から k を求める。

k ≦ -2 のときには m=3-k と置けば k^2+3k+9 = m^2+3m+9 なので
同様に出来る。
357354:03/08/09 15:34
>>355
言葉足らずでした。四辺は全て直線と考えてます。
>>353
x=(k±√(k^2+3k+9))/2=(2k±√{(2k+3)^2+27})/4
x整数より(2k+3)^2+27=n^2(nは自然数)
ここで、3kも2kも整数なので、kも整数。
よって|2k+3|は自然数。
m=|2k+3|とおくと、
m<nより、m+1≦n
n^2≧(m+1)^2=m^2+2m+1
n^2=m^2+27より
2m+1≦27
m≦13
m≦13でm^2+27が平方数になるのは、m=3、13のみ。
|2k+3|=3または13より、k=-8、-3、0、5
実際、この4通りの場合は、いずれも2つの整数解を持つ
>>357
四辺はもちろん線分(直線の一部)になるけども
折れ線で分割すれば四角形三つに簡単に分割できる。
と言いたいだけ

直線で分割するならば帰納法で
1回目の分割(二分割)は必ず片方は三角形になる。
(頂点を通れば両方三角形、通らなければ片方は四角形)
n回目の分割でも一つは三角形があるとすると
n+1回目の分割でも一片は三角形になる。
よって何回(有限回)分割しても一片は三角形になる。
頂点を通らない分割を残った三角形に続ければ
四角形の合併はもとの三角形に収束する。(いくらでも元の形に近づく)
これを四角形に分割できたと言うかどうかは君しだい
361354=357:03/08/09 16:04
>>359
ああぁぁ...自分は長方形しか頭に無かった...恥ずかしい...

>>360
>>これを四角形に分割できたと言うかどうかは君しだい
できたと言えると思います。
何故なら、出来ないと言うと微分・積分を否定する事になるので。

いやー勉強になりました。と言うか自分の頭の固さを思い知らされました。
362132人目の素数さん:03/08/09 16:16
>>352
置換積分って何?
解き方教えてください。
xyz空間で, -1≦x≦1, -1≦y≦1, 0≦z≦2, z≧x^2+y^2
を満たす領域に4頂点がある三角錐の体積の最大値はいくら.
>>363
なんだか、場合分け大杉。
素直に4/3だったりはしないんだろーなー
>>362
置換積分しないし, 不親切だが間違ってはいない解答.

(-2/3)( √(2-x) )^3 を微分するとちょうど √(2-x) になるから,
F(x)=(-2/3)( √(2-x) )^3とすると, 定積分の定義により,
求める定積分は
F(2)-F(0) = (2/3)( √2 )^3 = (4/3)√2
>>364
出題者です. たぶん4/3で正解. すくなくともその1.2倍以下.
簡単な説明はないでしょうか?
367132人目の素数さん:03/08/09 18:34
うわーw
すげっす。尊敬
368132人目の素数さん:03/08/09 21:51
(条件つき)期待値E(X|Y)を考えます。
E(X|Y)はYだけの関数ですよね?
ラグランジュの未定乗数法使えないの?
計算が楽にはならないだろうが、場合分けしなくてもよくなりそう。
>>368
そ。
与式の最大値、最小値を求めよという問題がよくあるんですけど
どのようにして解くんでしょうか?

例えば、あるサイトにあったんですけど
ab/4(a+b)の最大値を求めよって問題です。
どのようにするんですか?
どなたかおしえていただけないでしょうか。
よろしくお願いします。
372132人目の素数さん:03/08/09 23:29
統計学の質問ってスレ違い?
>371
定義域にもよるしケースバイケースだけど

ab/(4(a+b)) = 1/{4 ((1/a)+(1/b))}だから

(1/a)+(1/b)の最小値を求めるっちゅーこともある
>372
統計学なんでもスレッド
http://science.2ch.net/test/read.cgi/math/1012782106/
375132人目の素数さん:03/08/10 00:21
376132人目の素数さん:03/08/10 01:05
4次元空間だとどんな不思議なことがありますか?



377132人目の素数さん:03/08/10 01:17
X^Y=A
Y^X=B
X/Y=C

このときXとYをAとBとCを用いて表せ!!!
378132人目の素数さん:03/08/10 03:02
「ボールが12個あります。重さが違うボール(重いのか、軽いのかも不明)
が一つだけまざっています。天秤を3回使って重さが違うボールを当てなさい」

という問題です。お願いします
>>378
どこか他のスレで解決してたよ
探しな
>>378
添削してやる。出直して来い。

「ボールが12個(・∀・)アルYO!!。重さが違うボール(重いのか、軽いのかもフメーイ)
が一つだけマザーテいます。天秤を3回使って重さが違うボールを当ててくれYO」
四回要るだろう
383132人目の素数さん:03/08/10 10:05
ある無理数で、小数部分はあらゆるパターンが出てくるのですか。
例えば、00000001とか0000000000001とか。

問題ではないので答えはありません。

0<mx-n<kでm,nは自然数,kは任意の小さい数、xはある無理数の問題で
mxの小数部分がk(例えば10^-dとか)より小さいということになるから、そんなのできるか
疑問におもったのです。
384132人目の素数さん:03/08/10 10:13
10年後に教えてやるよ!
そんなことは無いだろう
0.010110111011110111110111111...
とか適当に構成してみると…
386math.1st ◆M9pCfogc9g :03/08/10 10:22
Re:>376 まずはあなたに4次元ベクトル場のrotの定義を試みてもらいたい。
ちなみに3次元ベクトル場F=(f,g,h)のrotFは、(h_y-g_z,f_z-h_x,g_x-f_y)である。
_xなどは偏微分。

Re:>385 質問は良く読んだ方がいい。

Re:>383 0.012345678900010203040506070809101112131415...
4次元ベクトル場のrotって何だよ
4次元テンソル場のrotだろ
388132人目の素数さん:03/08/10 10:45
>>386
無理数は循環小数ではないから小数部にあらゆるパターンが出るから
m=10^sにして、てきとうにkより小さい小数部にできるということ?
389132人目の素数さん:03/08/10 10:56
まじくだらんことだが気になったんで・・・

三角形ABCで∠A∠Bに対応する変がabで
a>bと∠A>∠Bは同値っていうのはなんでですかね?
だれか証明してくらはい。
390math.1st ◆M9pCfogc9g :03/08/10 11:05
Re:>389 転換法で示す。すなわち、
a>b⇒angle(A)>angle(B),a=b⇒angle(A)=angle(B),a<b⇒angle(A)<angle(B)の三つを示す。
a=bのときは、二等辺三角形の問題になる。
a>bのときは、頂点BからAに向かって直線を引いて、その交点をDとしてCD=CBなるようにする。
実際に図を描けばどう証明すべきかはわかるだろう。a<bのときも同様。
391132人目の素数さん:03/08/10 11:06
こんな問題(答えはわかりません)
s=Σ1/(10^(n(n+1)/2)=0.101001000100001...
循環小数ではないから無理数になるけど、何になるか?
392math.1st ◆M9pCfogc9g :03/08/10 11:09
Re:>387 テンソル代数に積を入れられるようだけど、
4次元ベクトル場にrotを導入するのは無理があるのかなぁ?

Re:>388 それはいちいち訊かなくてもすぐに分かると思うが。
393math.1st ◆M9pCfogc9g :03/08/10 11:13
Re:>391 これは代数的になるか超越的になるかというもんだいなのかな?
394132人目の素数さん:03/08/10 11:16
>>393
そうです。無理数でもいれこの分数で循環するのもあるし、代数式で表現できる
のもあるし、eみたいなのとか、π/2とか、何かで表現できないでしょうか。
395132人目の素数さん:03/08/10 11:19
>>390 わかった!センクス!
396math.1st ◆M9pCfogc9g :03/08/10 11:19
Re:>394 リュービルの定理
整数係数n次方程式は…
いちいち覚えてないから検索してみてくれ。
397387:03/08/10 11:19
>>392
ああごめん。よく読んでなかった。
4次元(以上或いは未満でも)ベクトル場Fのrot Fは∂iFj-∂jFiとして
結果はテンソルで一般化するのが普通。
398132人目の素数さん:03/08/10 11:42

おりょりょ、今思ったんだけど、解が整数の方程式なら素因数分解でとけるかも!
399132人目の素数さん:03/08/10 11:51
自然数a、bに対しaをbで割った商をq、余りをrとすると、
a、bの最大公約数とb、rの最大公約数が等しいことを証明せよ

という問題なのですが、解けそうで解けません
どうか教えていただけないでしょうか?
400132人目の素数さん:03/08/10 12:02
(x+y)=4、xy=3の時、x^2+y^2の値を求めなさい。

これの答えって10でいいんですよね。
>>400
OK
>>399
a,bの最大公約数Gはrを割り切り
b,rの最大公約数G'はaを割り切ることを示す。
GとG'の関係は?
402132人目の素数さん:03/08/10 18:35
x^2+6x+13の解は?
403132人目の素数さん:03/08/10 18:41
>>391
超越数です。
塩川宇賢 無理数と超越数 森北出版1999 
参照のこと。
404132人目の素数さん:03/08/10 18:53
>>402
まちがえた。
x^2+6x+13=0です。
>>404
x=-3±2i
406132人目の素数さん:03/08/10 19:51
全ての自然数nに対して (21n+4)/(14n+3) が既約分数であることを証明せよ

という問題を一から教えてください
帰納法使え
408132人目の素数さん:03/08/10 20:24
>>407
すみませんが、もうちょっと具体的にお願いしたいです
409132人目の素数さん:03/08/10 20:37
相異なる(n+1)個の整数がある。
これらの中から2つの数を選べばその差がnで割り切れるものが存在することを示せ。

という問題なのですが、これって推測してから帰納法なのでしょうか?
よく分からないのでご教授くだされば幸いです。
>>409
鳩の巣原理
>>409
n色の玉からn+1個を選べば、必ず同色のペアが存在するだろ?
これを引き出し論法とか鳩の巣原理とか言う。
何を「推測」するのかよく分からんけど
直接nの場合に示せるでしょ。
nで割った余りで分類していけばいい。
>>409
鳩ノ巣原理
nで割った余りは最高でもn種類
(n+1)個でn種以下なので同じ余りの組が存在する
415409:03/08/10 20:53
どうにか解けそうです
みなさんありがとうございました
41632人目の素数さん:03/08/10 20:57
>>406
3(14n+3)-2(21n+4)=1より
(14n+3)と(21n+4)の最大公約数は1
だから既約.
417132人目の素数さん:03/08/10 21:06
>>416
そうかなあ・・・
418132人目の素数さん:03/08/10 21:21
>>417
?????????
419417:03/08/10 21:24
>>418
すみません。よく分かっていないもので。
nの式が互いに素だと任意のnに対して値も互いに素なんですか?
もしよろしければ説明いただければと。
>>417
>>416の3(14n+3)-2(21n+4)=1という式は恒等式なんだぜ。
417の1をコピペしそこねた「32人目の素数」です.

>>417 >>419

分子と分母の公約数 d で約分するのだが
3 * 分母 - 2 * 分子 は d の倍数にしかならない.
それが1なんだから 公約数は1以外にない.

宿題: 全ての自然数nに対して (21n+5)/(14n+3) が既約分数であることを証明せよ
422417:03/08/10 21:31
任意のnに対してその式が成立するから、
(14n+3)と(21n+4)に共通因数が存在するとしたら、
その式は1になるはずない。

これでいいんでしょうか?
423132人目の素数さん:03/08/10 21:32
>>420,421
わかりました。ありがとうございました。
>>419
すまない. 宿題の意図に合わない出題をしてしまった. 改めて

宿題: 全ての自然数nに対して (14n+5)/(14n+3) が既約分数であることを証明せよ
夏ですね。
互助砲ドカーン
入り砲ズキューン
夏か、夏なんだな!そうか、夏か・・・。
X = g(x)
Y = f(x)
といった、XY座標平面というのに、媒介変数で表された曲線というものがイメージできないのですが、
どのように考えればよいでしょうか?
x=0 を代入してみる。すると点が1個定まるだろ。
x=1 を代入してみる。すると2個目の点がまるだろ。

これを(xに対し)全ての実数で行なってみて、
できた曲線がそれ。

こんなんでわかるかね。
                       道を 歩いている ようなものだ
>>430
なんとなくわかりました。どうもです。
433132人目の素数さん:03/08/11 13:36
  _, ._
( ゚ Д゚)
434132人目の素数さん:03/08/11 18:31
クソレスの数の方がおおいスレ発見
435132人目の素数さん:03/08/11 22:50
線型計画法の問題で
目的関数S=x+y+z
制約条件 2x+ 3y+ 2z≦12
4x+ 5y+20z≦40
20x+12y+15z≦60
x+ 2y+ 3z≦6
x≧0,y≧0,z≧0

この答えってx=1.714,y=2.143,z=0であってますか?
436132人目の素数さん:03/08/11 22:51
↑は目的関数の最大化の問題です
437132人目の素数さん:03/08/11 23:56
折り紙を1回折ったとき、1%の確率で折り方を失敗するとする。
1回でも折り方を失敗した折り紙は、捨てられるものとする。

以上の条件の下で、44マソ羽の鶴が完成するまでには、何枚の折り紙が捨てられることになるだろうか?
>>437
いっぱい。
>>437
99%が44万だからさ、440000/99くらい
1羽の鶴は何回折ればできるんだ
>>435
x=12/7。
y=15/7。
z=0。
S=27/7。
>>439
案外正解かも…
何で案外なんだろう・・・
>442が確率の意味を分かっていないから
445132人目の素数さん:03/08/12 00:38
∫[0〜(π/2)] 1/(4+5sin x) dx

解法お願いできませんか

答えはlog2/3だそうです。

お願いします
すみません。
もうひとつのスレッドで質問したのですが、
あっという間に流されてしまったので、再度こちらで質問させて下さい。

コーシーの平均値の定理がよくわからないのでその意味について質問して、
ラグランジュの平均値の定理とほとんどいっていることは変わらないという
レスをいただいたのですが、それでもよくわからないので、教えて下さい。

(そのまえに「ラグランジュの〜」というのは、(f(b)-f(a))/(b-a) = f'(c) というやつのことですか?)

関数が二つになると、なんかよくわからないです。
二つの関数f(x)とg(x)は、独立していて、特に関係はないんですよね?
ab間を結ぶ直線の傾きと、cでの接線の傾きとの比(?)というのに
どういう意味があるのか?です。イメージがわかないです。

447132人目の素数さん:03/08/12 00:43
半径がa(>0)の球に内接する直方体の表面積のさいだいちを求めよ
また体積の最大値をもどめよ
>>446
コーシーの平均値の定理で、g(x)をただの直線と考えれば、
平均値の定理が得られる。
449132人目の素数さん:03/08/12 01:01
>>441
ありがとうございます。もう放置されたかと・・・・。
今度昼飯でも奢りますよww
>>435
 28S
=28x+28y+28z
≦28x+28y+39z
=(20x+12y+15z)+8(x+2y+3z)
≦108。
>>446
a-b間を行ったり来たりすりゃいいじゃないか。ってイメージ。
>>447
球の中心を原点におき、直方体の各面をxy,yz,zxの各平面と平行にとっても
問題の一般性は失われない。
そうすると、直方体の頂点の座標は、(±x,±y,±z)(複号任意,x,y,zは正)
とおけ、表面積は8(xy+yz+zx)、体積は8xyzとなる。
これらの最大値をx^2+y^2+z^2=a^2という条件下で求める問題。
x^2+y^2+z^2-(xy+yz+zx)=(1/2)((x-y)^2+(y-z)^2+(z-x)^2)≧0より
表面積≦8a^2
(等号条件は、x=y=z=(√3/3)a)
また,相加平均≧相乗平均で
(x^2+y^2+z^2)/3≧(xyz)^(2/3)より
体積≦(8√3/9)a^3
(等号条件は、x=y=z=(√3/3)a)
453132人目の素数さん:03/08/12 04:15
なぜ、こちらは人気がないのか?
おんなっけがないからか?
454132人目の素数さん:03/08/12 04:16
マスコットキャラとして、亞里亞でも雇え!
こっちで遊ぶな
質問厨がくだらなくない問題だと思うっているから
20日で1スレなら昔のさくらスレなみ。
458132人目の素数さん:03/08/12 10:19
”高々2次の整式”の”高々”ってなんじゃぃ
なぜつける。


たかだか 【高高】

(副)
(1)3 2 (「たかだかと」の形で)
(ア)目立って高いさま。
「―と抱きあげる」
→鼻高々
(イ)声高(こわだか)に言うさま。
「声―と朗読する」
(2)2 0 どうみても。せいぜい。たかが。
「―百人が関の山だ」


三省堂提供「大辞林 第二版」より
459132人目の素数さん:03/08/12 10:43
10人の賢者がいました。
王がこの10人の賢者に金の冠をかぶせました。
賢者は皆、自分がどんな冠をかぶっているのか見えませんが、
他の9人がかぶっている冠は見えます。
王が言いました。
「私はお前たちに、金の冠か銀の冠のどちらかをかぶせた。
ただし、少なくとも1人は金の冠をかぶっておる。
自分のかぶっている冠は金の冠か、銀の冠かを当ててみせよ。」

しばらくした後、10人の中で1番賢い賢者が答えました。
「そうだ、私のかぶっている冠は、金の冠だ。」

さて問題です。
彼はなぜ、自分のかぶっている冠が金の冠だとわかったのでしょうか。
460132人目の素数さん:03/08/12 10:49
王がこの10人の賢者に金の冠をかぶせました。
>>458
>(2)2 0 どうみても。せいぜい。たかが。
せいぜい二次。勿論一次かも知れんし定数かも知れん。ってこと。
自分以外の賢者の冠が銀だったから。
463132人目の素数さん:03/08/12 11:48
.>>462
最初の条件で、「王がこの10人の賢者に金の冠をかぶせました。」というのがある。
464458:03/08/12 12:00
>461さま
アドバイスあがとうございます。
アドバイスの内容が自分はどうなんだと突きつけられてように感じました。
貴方はどう思いますか?といういくつかの問いは、なにが正しくなにがおかしいのか自分でわかっています。
おかしいのは分かっているが自分の気持ちがそう感じてしまうので悩んで相談した次第であります。



と"高々"でgoo教えてでひっかかった
No.618347 質問:彼女が外見的に満足できない・・・結婚すべきでしょうか
の恋愛相談の回答に対するお礼を張ってみる。
>>458
(2)の意味で用いられる場合、日常語だとどうしても
軽視する・馬鹿にする・相手にしない、などの感情が
入り交じった表現に聞こえてしまう。

(3) 多くとも。多く見積もっても。「少なくとも」の対義語。

というのを辞書に加えて欲しいところだ。
466132人目の素数さん:03/08/12 14:52
C(・,・)で二項係数(組み合わせの数)を表すとします。
pが素数、qがpと素な正の整数、eが正の整数のとき、
 C((p^e)q,p^e)≡q(mod p)
であることを示してください。よろしくお願いします。m(_ _)m
>>466
(x+1)^(qp^e)を(Z/pZ)[x]で2項展開してx^(p^e)の係数を比較。
468132人目の素数さん:03/08/12 15:04
>>467
466です。レス、ど〜もありがとうございます。
(Z/pZ)[x]で二項展開って、どうやるんでしょう?
>>459
この問題は面白いよ
470132人目の素数さん:03/08/12 15:25
>>459
2人の愚者がいました。
王がこの2人の愚者に金の冠をかぶせました。
賢者は皆、自分がどんな冠をかぶっているのか見えませんが、
他の愚者がかぶっている冠は見えます。
王が言いました。
「私はお前たちに、金の冠か銀の冠のどちらかをかぶせた。
ただし、少なくとも1人は金の冠をかぶっておる。
自分のかぶっている冠は金の冠か、銀の冠かを当ててみせよ。」

しばらくした後、愚者の中で賢い方の愚者が答えました。
「そうだ、私のかぶっている冠は、金の冠だ。」

∵もし私の冠が銀なら、少なくとも1人は金の冠をかぶっていることから、
相手の愚者は、 自分の冠が金であることが直ぐにわかる筈だ。
しかし、相手の愚者は何も言わない。従って、私の冠は金だ。

これを3人,4人,…10人と増やしていけば良いね。
471132人目の素数さん:03/08/12 15:45
>>1-470
番号を青色にするにはどうすればいいのですか?
473132人目の素数さん:03/08/12 16:33
t*sin at のラプラス変換で
-d/ds[a/(s^2+a^2)]=2as/(s^2+a^2)^2 となるらしいんですが
途中の経過の式がわかりません
式お願いできないでしょうか
>>459
この手の問題を、疑問なしに面白がるやつは、論理パズルに毒されすぎた
アフォなので、逝っちゃってください。

まず、疑問はあるにせよ問題として成立させるためには、時間軸を
「判断するステップ」でスライスしてやるなんらかの設定が必要なんだが、
(わかったかどうか質問して手を挙げさせる、ということを10回繰り返す、とか)、
たとえそういう設定をしても、全員が「このような状況の時は全員こういう
パターンで思考するに違いない」という論理パズルおたく的思考回路の
持ち主であることが前提にないと、やっぱ問題として成立しないでしょ。

「判断可能性を判断材料にする」というロジックについて根本的な疑問を
投げ掛ける問題、という位置づけで面白がるならいいけど。
>>474
アフォはお前だな。もう少し問題を吟味しなさい
お前がいぶかっているいくつかはきちんと設定されている
>>475
( ´,_ゝ`)ハイハイ
突っ込む気もおきな(ry
478付かぬ人:03/08/12 16:57
付かぬ事をお尋ねしますが、『微分方程式』って『微積』を勉強してからじゃないと理解できないのでしょうか。
めちゃめちゃレスが早いな。わざわざ二度も>476=477
>>478
と〜ぜんそうです。
ついでにいうと、最低限の理論(常微分方程式の入門)でさえ、
解析の基礎(いわゆる微積)のみならず、線型代数も使います。
微分方程式なんかで、線形という言葉が出てきますが線形とは
どういう意味なんですか
482132人目の素数さん:03/08/12 17:04
t*sin at のラプラス変換で
-d/ds[a/(s^2+a^2)]=2as/(s^2+a^2)^2 となるらしいんですが
途中の経過の式がわかりません
式お願いできないでしょうか
483132人目の素数さん:03/08/12 17:06
@
3辺の長さがx、x+2,98−xの三角形がある。
(1)xの値の範囲を求めよ。
(2)この三角形が直角三角形になるときのxを全て求めよ。
A
半径1の円に内接する正十角形ABCDEFGHIJがある。
(1)AB^2+EI^2+IA^2の値を求めよ。
(2)この正十角形の10個の頂点から異なる2点を選んで結び
線分を作る。そのような線分の長さの平方を全て考える時
それらの平均の値を求めよ。
B
3つの整数の組(a,b,c)が与えられていて、その3つの数の和は正であり、a,b,cの
うち、少なくとも1つは負であるとする。このとき、次の書き換え法則Aを考えると、
事柄Bが成り立つ。


書き換え法則A:
「3つの数のうちに負の数があれば、その数を他の2つの数に加え、その数の符号
を正に変える。例えば、aが負であるとき、(a,b,c)→(-a,b+a,c+a)
と書き換える。

事柄B:
書き換え法則Aを何回か繰り返して適用すると、3つの値は全て0以上になる。

設問:
事柄Bが成り立つことを証明せよ。


C正四面体をひとつの平面で切った切り口が正方形となり、その面積が1であるとき
正四面体の体積を求めよ。
(東京にある進学校および大学付属高校の入試問題)
484132人目の素数さん:03/08/12 17:06
お願いできました
>>473>>482
なぜレスが付かないか考えてみてください。
式の書き方が曖昧過ぎます。
その曖昧な式を解読して、答えてあげようという気が起こらないのです。
解答者のことも少しは考えたほうが良いと思います。
486132人目の素数さん:03/08/12 17:28
>>485
はい。
ラプラス変換に L[t*f(t)]=-d/dsF(s)  という式があります。
           L[f(at)]=(1/a)F(s/a)  

t*sin at をこの式に入れて解くと  t*sin atのラプラス変換 = 2as/(s^2+a^2)^2
という解答になるらしいのですが、途中経過の式が載っていなくて
わかりません。
途中式の経過をお願いします
487132人目の素数さん:03/08/12 17:30
堤さやかちゃんの引退記念作です。
これは絶対見るしかないでしょう。
甘えたしゃべりかた、小さな身体に大きなオッパイ、そしてこの顔。
どれをとっても特A級!こんな子がAV女優だったなんて信じられませんね。
無料ムービーを観てね。
http://www.exciteroom.com/
>>486
sin(at)={exp(iat)-exp(-iat)}/2i
のラプラス変換を求めればいいんじゃないの。
>>486
マルチうざい 
>>489
それはしょうがないと感じないのか?
心の狭いやつめ
>>490
やはりマルチは良くない。
他スレで煽られて去らざるを得なくなった場合には、その旨断れば済むこと。
そうすれば、誰もマルチと言わずにちゃんと相手してくれる。

>>486は、煽られた訳でもないのに、元スレに断りなく他スレに同じ質問を立てたのだから、
マルチとして放置されても仕方ない。
持ち物を自慢したのだからシカトされても仕方ない
493132人目の素数さん:03/08/12 19:37
あのう〜 >>466>>468 分かる方いたらよろしくお願いします。
494付かぬ人:03/08/12 20:07
>>480
レスありがとう。
495132人目の素数さん:03/08/12 20:13
数列を a_n=1/n とします。
極限が lim_[n→∞]a_n = 0 であることを 数列の収束の定義に基づいて
証明したいのですが、ある本につぎのようなことが書いてありました。

まず、∀ε>0をfix
次に、アルキメデスの原理より

∃N∈{自然数全体} s.t. N+1 > 1/ε ・・・(*1)
 
⇒∃N∈{自然数全体} s.t. ε > 1/(N+1)

⇒∀ε>0 , ∃N∈{自然数全体} ; n > N ⇒|1/n - 0|= 1/n ≦ 1/(N+1) < 1/ε ・・・(*1)


ここで疑問が二つあるのですが、

一つめは、(*1)で N+1と取っていることについてです。
アルキメデスの原理なら N でもよいはずだし、証明も問題なさそうに思います。
なぜN+1をとっているのでしょうか?

二つめは、(*2)で1/n ≦ 1/(N+1)となっているが、ここは 1/n < 1/(N+1)
(あるいは 1/n < 1/N )でもよいのではないか。

というものです。
どなたか、教えてください。


496132人目の素数さん:03/08/12 20:13
>>459の問題は論理的思考能力を試すいい問題だと思う。
ラプラス変換って、どういう用途に使うの?
498ED:03/08/12 20:23
40人のクラスで、電車通学の生徒が25人、バス通学の生徒が18人である。
電車通学だけの生徒が9人のとき、どちらも利用しない者は何人か。
という問題なんですけど、お手上げです。答えは13人らしいんですけど、全然
わかりません。教えてくだい。
>>491
だから?それだけ?
ただ言うだけ?
500get
501132人目の素数さん:03/08/12 20:29
25-9=16
18-16=2
40-(25+2)=13
502ED:03/08/12 20:30
>>501
39
503132人目の素数さん:03/08/12 20:32
>>496
この問題って、各賢者が他の人がどれくらいの時間でどれくらい
論理的推論が行えるかということを知っているという前提がないと
解けないよな。
504132人目の素数さん:03/08/12 20:32
あの〜∫1/(x*√(1+x^2) dx お願いします。
置換積分なのか、それとも部分積分なのかわかりません。
よろしくお願いします。m(_ _)m
505132人目の素数さん:03/08/12 20:33
はっきり言ってかなり映像の質は悪いです。
しかしながらこの可愛らしい女子校生の顔を見ているだけで
勃起してしまうのは私だけでしょうか?
処女太りの体形もマニアにはたまらないでしょうな。
さてさて?のなかにはどんな数字が入るのでしょうか?
いえいえそれは絶対いえません。
無料ムービー観てね(当然モロみえ)
http://www.cappuchinko.com/
506132人目の素数さん:03/08/12 20:33
ラプラス変換に L[t*f(t)]=-d/dsF(s)  という式があります。
           L[f(at)]=(1/a)F(s/a)  

t*sin at をこの式に入れて解くと  t*sin atのラプラス変換 = 2as/(s^2+a^2)^2
という解答になるらしいのですが、途中経過の式が載っていなくて
わかりません。
507トップエリート街道さん ◆BIG6e4aEMg :03/08/12 20:33
>>493
係数で、pの倍数は0とみなすという感じで考えるといい。
(x+1)^(p^eq)={(x+1)^(p^e)}^q≡(x^(p^e)+1)^q
=x^(p^eq)+・・・+qx^(p^e)+1
普通に展開すると、x^(p^e) の係数はC(p^eq,p^e)だから、これと比べる。
508132人目の素数さん:03/08/12 20:41
>>495
2番目の(*1)は(*2)の間違い?
あと1/n ≦ 1/(N+1) < 1/εじゃなくて1/n ≦ 1/(N+1) < εな。

(*1)でN>1/εとして(*2)でn≧N⇒…としても一緒だよ。
著者がn>N⇒…と書くのを好んでいるとかじゃないの?
>>503
解けるかどうかは別としてそれは問題を読み間違えている
>>504
y=√(1+x^2)で置換するとよさげ。
ちゃんと計算したわけじゃないけど。
511132人目の素数さん:03/08/12 20:53
   1  0  -1
A= 0  -1   0
  -1  0   1

固有値は計算した結果、t=2、0、-1となりました
固有値に対応する行列Aの長さ1の固有ベクトルを求めよという問題なんですが、
長さ1のという意味が分かりません。ただ単に固有ベクトルを求めればよいのでしょうか?
よろしくお願いします。
>>511
ベクトルの長さって…高校生でもわかるやん
513トップエリート街道さん ◆BIG6e4aEMg :03/08/12 20:58
>>511
あるベクトルが固有ベクトルなら何倍しても固有ベクトルだよ。
だから長さを指定しないとたくさんある。
長さを1にすれば、向きのちがいの2つだけになる。
514132人目の素数さん:03/08/12 21:00
>>508

>2番目の(*1)は(*2)の間違い?
>あと1/n ≦ 1/(N+1) < 1/εじゃなくて1/n ≦ 1/(N+1) < εな。
そうです。

>著者がn>N⇒…と書くのを好んでいるとかじゃないの?
この本ではこう定義してありました。

>(*1)でN>1/εとして(*2)でn≧N⇒…としても一緒だよ。
…以降がわかりませんが、
(*1)でN>1/εとして(*2)でn≧N⇒|1/n - 0|= 1/n ≦ 1/N < ε
こういうことですか?

それと、
∃N∈{自然数全体} s.t. N > 1/ε ・・・(*1)
⇒∃N∈{自然数全体} s.t. ε > 1/N
⇒∀ε>0 , ∃N∈{自然数全体} ; n > N ⇒|1/n - 0|= 1/n < 1/N < ε ・・・(*1)
これはどうでしょうか?
515132人目の素数さん:03/08/12 21:04
>>504
(1/3)x^3+C かな
>>514
本質的に全部一緒だよ。
ε-N論法の理解で躓いているわけじゃないから安心しろ。
どうしても気になるなら≧バージョンから>バージョンを導くとかしてみれば?
517132人目の素数さん:03/08/12 21:07
>>512>>513ありがとうございます。固有ベクトルを計算してみたんですが

t=2のとき         t=0のとき         t=-1のとき
         1            1                1
p1=1/√2 ( 0 )  p2=1/√2 ( 0 )     p3=1/√5 ( 0 )
        -1            1                2

となったんですが、これで合ってますでしょうか?
518132人目の素数さん:03/08/12 21:09
>>509
あなたの答えは?
519132人目の素数さん:03/08/12 21:10
504 名前: 132人目の素数さん 投稿日: 03/08/12 20:32
あの〜∫1/(x*√(1+x^2) dx お願いします。
置換積分なのか、それとも部分積分なのかわかりません。
よろしくお願いします。m(_ _)m

515 名前: 132人目の素数さん 投稿日: 03/08/12 21:04
>>504
(1/3)x^3+C かな

((1/3)x^3+C)'=1/(x*√(1+x^2))だそうです。
520132人目の素数さん:03/08/12 21:10
サンプルならあるよ
http://www.k-514.com/sample/sample.html
>>514
{自然数全体} では "自然数全体" というものが元として入っている集合の意味になるな。
すると N は自然数全体からなる集合そのものと言うことになる。
522132人目の素数さん:03/08/12 21:11
>>510>>515
ありがとうございます。導けました。
523トップエリート街道さん ◆BIG6e4aEMg :03/08/12 21:15
>>517
t=-1 のときが変。
524ラ・サール高2(理系2位):03/08/12 21:21
>>504
(1/3)x^3+C
ラサールもレベル落ちたな…
526132人目の素数さん:03/08/12 21:27
∫範囲をI=[0,∞)
ラプラス変換
L[f(t)]
= ∫_I dt exp[-s*t] f(t)
=F(s)
が定義ですよね.
すると,F(s)をsで微分すると,
dF(s)/ds
=(d/ds)∫_I dt exp[-s*t] f(t)
=∫_I dt (d/ds)exp[-s*t] f(t)
=∫_I dt −t*exp[-s*t] f(t)
=−∫_I dt exp[-s*t]{t*f(t)}
=−L{t*f(t)}

L{t*f(t)}=−dF(s)/ds

527132人目の素数さん:03/08/12 21:27
L[f(a*t)]
= ∫_I dt exp[-s*t] f(a*t) 
(★)a*t = y ⇔ d(a*t) = dy ⇔ a*dt = dy, もし a>0 → I=[0,∞)
= ∫_I (dy/a) exp[-s*(y/a)] f(y)
= (1/a)*∫_I dy exp[-(s/a)*y] f(y)
: 定義L[f(t)]= ∫_I dt exp[-s*t] f(t)=F(s)

= (1/a)*F(s/a)
528132人目の素数さん:03/08/12 21:28
----------------------------------------------
L[f(a*t)]
= ∫_I dt exp[-s*t] f(a*t) 
(★)a*t = y ⇔ d(a*t) = dy ⇔ a*dt = dy:
もし a=-|a| <0 → I'=(-∞,0]
= ∫_I' (dy/-|a|) exp[s*(y/|a|)] f(y)
= (1/|a|)*∫_I dy exp[(s/|a|)*y] f(y)

a<0だとこの∫は発散してラプラス変換が存在しないので,
a>0のときだけ考えられる.
 t*f(t)でf(t)=sin(a*t)と考えて,t*sin(a*t)のラプラス変換を
考える.
529132人目の素数さん:03/08/12 21:28
L[e^(a*t)] = 1/(s-a) : a=i*b

L[e^(i*b)]
= L[cos(b*t)+i*sin(b*t)]
= L[cos(b*t)]+i*L[sin(b*t)]
= 1/(s-i*b)
= s/(s^2+b^2) + i*{b/(s^2+b^2)}

L[cos(b*t)] = s/(s^2+b^2),
L[sin(b*t)] = b/(s^2+b^2)          
530132人目の素数さん:03/08/12 21:28
よって,
L[sin(a*t)]
= ∫_I dt exp[-s*t] sin(a*t)
d/ds{ - L[sin(a*t)] }
= - ∫_I dt exp[-s*t] (-t)* sin(a*t)
= ∫_I dt exp[-s*t] t*sin(a*t)
= d/ds{ a/{(s^2+a^2)}
= -a*2*s/(s^2+a^2)^2

2つは同じ.
よって,
L[t*sin(a*t)] = 2*a*s/(s^2+a^2)^2
>>519
だったらおまえやれば?
>>526-530
ありがとうございました マルチ気をつけます。
すいませんでした。
533132人目の素数さん:03/08/12 21:36
>>523
あれ?間違えてましたか。もう一度計算してみますので
間違っていれば突っ込んでください。

t=-1の時、(A-tE)x=0を満たす固有ベクトルを求めると
 2x-z=0
 x-2z=0
よりz=x
        1
p3=1/√2 ( 0 )
        1
これで合ってますでしょうか?
534132人目の素数さん:03/08/12 21:38
定義に従えば以上.
もしくは,ラプラス変換を⇔で考えると
sin(a*t) ⇔ a/(s^2+a^2)
t*f(t) ⇔ -dF(s)/ds
t*sin(a*t) ⇔ d/ds {-a/(s^2+a^2)} = 2*a*s/(s^2+a^2)^2
>>531
質問者の方で解決したみたいだしもういいでしょ。
しかし質問者も(1/3)x^3+Cとか言ってたら悲惨…

ん?実は>>504の閉じカッコが1つ足りないのが罠とか?
536トップエリート街道さん ◆BIG6e4aEMg :03/08/12 21:39
>>533
x=z=0 で、yは任意
537132人目の素数さん:03/08/12 21:49
>>519
∫dx1/[x*√(1+x^2)]
: √(1+x^2) = y,dy=x*dx/[√(1+x^2)]=x*dx/y

∫dy(y/x)*1/[x*y]

∫dy1/x^2

∫dy 1/[1-y^2]

∫dy (1/2)*{ 1/(1-y) + 1/(1+y) }

(1/2)*{-log(1-y) + log(1+y)}
=log{(1+y)/(1-y)}^(1/2) :y=√(1+x^2)
・…
こんな感じでしょうか?

538511:03/08/12 21:49
>>536
なぜそうなるんでしょうか?
>>537
惜しい。
4個目の式の符号が…
540132人目の素数さん:03/08/12 22:13
数学板には、誰も>>459を論理的に説明できる奴が
541540:03/08/12 22:38
いないのか。
学歴板>>数学板
ですね。
>>540-541
>>470さんが解いてんじゃん。
543132人目の素数さん:03/08/12 22:48
>>504
t=x+√(1+x^2) とおけばいい。
x=(t^2-1)/(2t) に注意すれば、
log|t^2-1|+C
544132人目の素数さん:03/08/12 22:57
>>542
愚者じゃなくて、賢者じゃないと駄目なんじゃないの?
545132人目の素数さん:03/08/12 22:59
>>537
符号だけ・・・
最終形も符号だけ・・・
546132人目の素数さん:03/08/12 23:02
>>511
連立方程式を解くとき係数だけを取り出して行列の形にしてから、
行列を掃き出し方で変形して答えを求めるわけだけど、
固有ベクトルを求めるときにも同じことが言える。
つまり、固有ベクトルを求めるときx,y,zなどという変数を持ちだして
>2x-z=0
>x-2z=0
などという新たな方程式を解く事をせず、行列
2 0 -1
0 0 0
-1 0 2
を掛けたら0ベクトルになるようなベクトルを求めればいいわけだから、
t=-1 のときの固有ベクトルは(0,1,0)' ('は転置を表す)ということが簡単にわかる。
547540:03/08/12 23:16
>>542
数学板的には、>>470が答えということでOK?
548511:03/08/12 23:18
>>546
連立方程式を解くことばかり考えてました。
とても参考になりました。ありがとうございました。
>>470
の答えじゃわかんないよ〜(;−;)。
>>544
愚者というかなんというかその最終的に自分は金の冠を被ってるっていった香具師は
自分以外の人間がだれも金の冠を被ってると申告しなかったことを情報として利用して
解答したわけでその情報が情報として価値のあるもの、すなわち十分に洗練された
思考の結果なお申告できなかったという仮定の上でなので他の人間が完全な愚者と
いう仮定だとどうしようもない。まあ、いづれにせよこの問題は問題文にない仮定を
解答者が勝手にいろいろ導入しないと解けない(というか解答も微妙にインチキ)ので
数学パズルというよりはちょっとウィットにとんだナゾナゾぐらいの気持ちでみとかないと
楽しめない。楽しむ必要もないけど。
551446:03/08/12 23:39
レスありがとうございます。

>>448
なるほどです。
証明で、媒介変数を使っていた理由がちょっとわかりました。

>>451
すみません。
よくわからないです。
552132人目の素数さん:03/08/12 23:46
「nをある正の整数とする。次の不定方程式
 k_1 + 2*k_2 + 3*k_3 = n …(1)
を満たす0以上の整数 k_1, k_2, k_3 に対して 
 f(k_1,k_2,k_3) = (k_1+k_2+k_3)!/(k_1!*k_2!*k_3!) …(2) 
が最大値をとるときの k_1, k_2, k_3 を求めよ。」 
という問題なんですがよろしくお願いします。
>>504
√(1+x^2) = t とおいて置換積分すっと、∫1/(t^2 - 1) dt
= (1/2)∫1/(t - 1) - 1/(t + 1) dt
既に解決した問題に頑張ってレスしてるヤツがいるな…
555132人目の素数さん:03/08/13 00:48
555
>>554
解けもしないでチャチャ入れるのが精一杯だな(ry
>>556
自己紹介はいいって。
558 ( ´,_ゝ`)プッ! :03/08/13 11:05
>>557
自演まちゅりか?
559132人目の素数さん:03/08/13 11:58
558 : ( ´,_ゝ`)プッ!  :03/08/13 11:05
>>557
自演まちゅりか?
5602のバカ女(T_T):03/08/13 12:06
●夏休みの宿題●
《 1 2 3 4 5 6 7 8 9 2 =50》
上の数字間に+−×÷や括弧をいれて50をつくれ
全然わかりません…。
561132人目の素数さん:03/08/13 12:22
自分でやれよボケ
>>561
できないんだな?
5632のバカ女:03/08/13 12:29
本当にわからないんです。
宿題は自分でやれ。ただのパズルだろ。
565132人目の素数さん:03/08/13 12:31
わかるわからんのはなしじゃないし。
566132人目の素数さん:03/08/13 12:32
こんなのいくらでも答えあるだろう。
例えば
1-2+3+4+5+6+7+8+9×2=50

これじゃつまんないから四則全部使った奴を考えてみろ。
5672のバカ女:03/08/13 12:33
何度も自分でやったんですけどほんと、ぜんっぜんわかんないんです。
中学生のこんなパズルがわかんなくて私って
本当にばかだし…
568132人目の素数さん:03/08/13 12:34
バカなのはみれば判るから一々書くなよ、イライラする。
5692のバカ女:03/08/13 12:36
いくらでも答えがあるんだぁ。
てか四則全部使ったのなんて考えられないんスけど
幾らでもあるから自分でやれっつってんだが・・・。
571552:03/08/13 12:54
お願いします。
572132人目の素数さん:03/08/13 12:55
(1*2*3*4+5+6-7+8*9)/2
573132人目の素数さん:03/08/13 13:26
1*2*3+4*(5*6)-8+9*2
5742のバカ女:03/08/13 13:30
みなさんスゴすぎですね('▼`)助かりました。
ありがとーございまちたっ☆
こんなの数学力関係ないだろ。
576132人目の素数さん:03/08/13 18:54
2のバカ女さん降臨期待age
5772のバカ女:03/08/13 18:55
呼んだ?
呼んでない
579132人目の素数さん:03/08/13 18:59
お色気たっぷりなんですがどこか可愛らしさが漂う綺麗な人妻です。
スレンダーなボディーが更に色気を倍増させている気がします。
じっくり見つめながらフェラをし、小さめのオマンコに男根を導いていきます。
こういう年の取り方をしたいですね。
無料ムービーからどうぞ
http://www.excitehole.com/
580132人目の素数さん:03/08/13 22:30
・ちょっと軽い問題(簡単だけどたまにわからないやつがいる)
@¬(P→O)⇔?
A一次方程式ax+b=0を解くとa=o∧b=0の時の解はx∈Rですよね
 ここで、恒等式の定義は恒等式とは,「等式に含まれているある文字に任意の文字を代入しても,
その等式の両辺の値が存在する限りつねになりたつ等式のこと」とか参考書には書いてありますよね
じゃあax+b=0は恒等式なんですか?
 なんでこんな矛盾が発生したか説明せよ
581132人目の素数さん:03/08/13 22:58
マルチポストですがすみません
わからない問題スレが激しく荒らされているので・・・
良い方法があったら教えてください。

実数の関数で
f( a , b ) = c

a > 0 かつ b > 0 の時 c は正
a > 0 かつ b < 0 の時 c は負
a < 0 かつ b > 0 の時 c は負
a < 0 かつ b < 0 の時 c は負
それ以外の時(要するにa b かどちらかが0の時)できれば 0 が望ましい

となるような関数を「定数」「和」「差」「積」のみで作りたいのですが、そういった関数は作れるでしょうか?
とりあえず、これに絶対値を加えると

f(a,b) = ( a + b + | a - b | ) * a * b

というのができたのですが、使用している計算機の命令セットの都合上できれば絶対値も使いたくないく、なんとかならないかと考えているのですが
良い方法があったら教えてください。

ちなみにこれは a b をデカルト座標 x y の上にマップして45度回転して、y座標の絶対値を取り、さらに45度回転すると x 軸にともに負の時のみ負になるように
できたので、ここに a * b を掛けて作りました。
582直リン:03/08/13 22:59
>>581
>「定数」「和」「差」「積」のみで作りたいのですが
つまり、fはa,bの多項式ということ。
題意を満たす多項式は存在しない。
>>581の意図はプログラムすることらしいから、素直にifを使うべき
584教えてくださいませ:03/08/13 23:54
統計学でもいいですか?ダメなら無視してください。

ある板で2chネラ数の地域ごと(都道府県とか)の統計をとってまして、
その集計から各地域毎の2chネラ率を出したいのですが、単純に、2chネラ数
を地域の人口で割って出したものを比較していいのでしょうか。
分子は数10人とか数人の場合が多く、分母はおもに数十万単位ですが、
人口が少ない地域(10万人とか)は多い地域(90万人とか)に比べて、
2chネラ率が必要以上にハネ上がってしまうような気がして、これで
いいのかなと思ったものですから。

変な質問ですみません。
数学とか統計とか全く分らない素人なので…

>>584
打席数の少ない選手の打率と打席数の多い選手の打率を考えてみよう
>>584
基本的にはその方法でいい。
ただし、統計数が少ないと、当然ながら誤差が大きくなるけどね。
>>584
別の言い方をすれば、「統計は嘘をつくのに最良の道具」だからそれで十分。
統計学そのもの自身の価値と統計で得た情報の価値とは無関係。
588580:03/08/14 02:13
誰か答えてみろよ。1はさすがに簡単だが、2はわかる人少ないかな?
589132人目の素数さん:03/08/14 02:15
0x+0=0は恒等式
>>588
答えてみろよって・・・問題の意味すらプーなんだが・・・工房?
591132人目の素数さん:03/08/14 02:45
>589
いや、でもそれは最初の一次方程式の特別な場合だけで、ひとつの式が方程式
でもあり恒等式でもあると考えられることは矛盾しないか?答えになってないと思う。
まあ、俺がいえることは、aが0でないときは解は-b/a、aが0かつbが0でないときは解は空集合の要素
ってことだけだな
@はP∧¬Oだと思う。
0x+b=0は一次方程式ではない
∂^2u/∂t^2=c^2(∂^2u/∂x^2)・・・(1)
という問題で解いを u(t,x)=T(t)X(x)とおいて解き
X(x)=定数*sin(nπx/l)
T(t)=An*cos(c/l)*nπt+Bn*sin(c/l)*nπt
という答えになるところまではわかるのですがその後
(1)は線形の方程式であるから、解の任意の線形結合は、また解となる。よって
u(t,x)=納n=1,∞]{An*cos(c/l)*nπt+Bn*sin(c/l)*nπt}*sin(nπx/l)
というのが解りません。どうしてこうなるか教えてください
594593:03/08/14 03:14
境界条件は,u(t,0)=0,u(t,l)=0です
595132人目の素数さん:03/08/14 03:25
2+5は7
596132人目の素数さん:03/08/14 03:26
素数を見つける方程式をだれかつくってくれ。
597132人目の素数さん:03/08/14 03:57
>>598
線型の方程式ではある解に対して、その定数倍も解になるし、
複数の解が存在する場合は、それらの和も解となるという性質がある。
u(t,x)=T(t)X(x)={An*cos(c/l)*nπt+Bn*sin(c/l)*nπt}*sin(nπx/l)
が任意の正整数nについて(1)の解になるわけだから、
これらをすべて加えたものも当然解になる。
598597:03/08/14 03:59
いかん眠い・・・。
>>598じゃなくて
>>593
599ちんこ成長薬:03/08/14 04:00
(σ・∀・)σゲッツ!! 600
>>580
問(2) なんでこんな矛盾が発生したか
答  580が参考書に書いてある定義を理解できていないから
>>601
せいか〜い
603教えてくださいませ:03/08/14 11:58
>>585,586,587

有難うございました。

結局目的は、日本に散らばっている2chネラの、地域毎の割合の相対的な
度合い〔都道府県別のランク)を知りたかったのですが、サンプル数が少
ないと誤差も大きくなるということですか?
つまり、例えば東京都内の場合、世田谷区は人口82万人に対して、集計で出た
2chネラ数は62人、また、荒川区は人口18万人に対して2ちゃネラ数9人で
したが、このように分母に対して分子があまりに小さいもの同士を比較対照して
もあまり意味が無いということなのでしょうか。

>統計数が少ないと、当然ながら誤差が大きくなるけどね。

というのはそういうことなのでしょうか。
すいません・・夏休みの宿題で自由研究があるんです
で 昆虫採集出そうと思ってるんですけど なかなか虫が見つかりません
どうすればいいのでしょうか?
釣れるか!つれるはずねぇ('A`)
>>604
あのな、ここは宿題をやってもらうスレじゃないんだよ
ほかの板なら教えてもらえるかもしれないけどな
かわいそうかもしれないがキミは放置されるよ
>>606
うるさいです しんでください
ちょっとだけだからいいじゃないのかよ ばか
608 ◆tq7XG53QPc :03/08/14 13:00
>>607
ここは数学板
数学以外の話題は板違い
609132人目の素数さん:03/08/14 15:06
三角形に内接する四角形で面積が最大となるものを
定規とコンパスだけで作図する方法を教えて下さい。
610132人目の素数さん:03/08/14 15:08
堤さやかちゃんの引退記念作です。
これは絶対見るしかないでしょう。
甘えたしゃべりかた、小さな身体に大きなオッパイ、そしてこの顔。
どれをとっても特A級!こんな子がAV女優だったなんて信じられませんね。
無料ムービー観てね
http://www.exciteroom.com/
611GoGO718 :03/08/14 15:22
場合の数 組合せで
「nCr=(n-1)C(r-1)+(n-1)Cr ただし1≦r≦n-1,n≧2 わかりづらいので括弧つけときました
この性質が成り立つことを、組合せの考えを用いて説明せよ。」
この問題ずばりどう説明すればよいですか?


612GoGO718 :03/08/14 15:24
森久美子ちゃんの引退記念作です。
これは絶対見るしかないでしょう。
豚みたいなしゃべりかた、大きな身体に大きなオッパイ、そしてこの顔。
どれをとっても特S級!こんなババアがAV女優だったなんて信じられませんね。
無料ムービー観てね
http://www.exciteroom.com/

>>611
1〜nの番号からr個取り出す組み合わせnCrを考える。
@1を含んでいる組み合わせと、A1を含んでいない組み合わせの2つの和として計算する。
 @=既に1が選ばれているので、2〜nのうちr−1個選ぶ組み合わせ=(n−1)C(r−1)、
 A=1つも選ばれていないので、2〜nからr個選ぶ組み合わせ=(n−1)Cr
である。
従って、nCr=@+A=(n−1)C(r−1)+(n−1)Cr■
614newton:03/08/14 16:08
方程式はある文字についての「条件」、恒等式は「命題」。それを理解してないか矛盾が発生した理由が説明できる。
まあ、数学基礎論やってれば余裕だな。
615newton:03/08/14 16:09
↑それを理解していれば矛盾が発生した理由がわかる
616132人目の素数さん:03/08/14 16:11
>組み合わせnCrを考える
不適切な表現です。
617132人目の素数さん:03/08/14 16:12
漸化式
x_(n+1)=((x_n)^2+k^2)/2x_n 
kは自然数

解けますか?
618132人目の素数さん:03/08/14 16:26
図形の合同の証明をするときに
「仮定より〜」とか、「合同な三角形の対応する辺と角は等しいので〜」とか
書かないとダメですか?
>>618
書かないと証明になりませんw
従属変数と独立変数の違いについて教えてください

>>620
違いも何も・・・独立な変数とそれに従属する変数のこと。
従属変数と独立変数の違いについて教えてください

違いも何も・・・独立な変数とそれに従属する変数のこと。
624 ◆BhMath2chk :03/08/14 17:40
>>617
y(n)=(x(n)+k)/(x(n)−k)。
>>624
どうやったらその変形思いつくんでつか?
626132人目の素数さん:03/08/14 18:17
>>583
もしよろしければ多項式でできない理由を教えていただけますか?

ちなみに 命令セット中に if は無いので使いたくても使えません。(^^;
絶対値とmax min で大きいほうを選択とかといったものはあるのですが、パイプラインの都合上これは使いたくなくて・・・
627132人目の素数さん:03/08/14 18:23
arg1/z=-argz+360゜×nで、式変形がんばってやってでたことはでんですが、何故
角度が反転するのか教えて下さい
628132人目の素数さん:03/08/14 18:30
>>624
yってなんすか
>>626>>581
関数fは「定数」「和」「差」「積」のみで作られているから多項式。

多項式fは四条件を満たすことから、f(0,b)=f(a,0)=0も満たす。
従って、定数項=0、かつ、aだけの項またはbだけの項はない。
そこで、多項式gを用いてf(a,b)=abg(a,b)と書ける。

このとき、fの四条件から、
 a>0∧b>0⇒g(a,b)>0  a>0∧b<0⇒g(a,b)>0
 a<0∧b>0⇒g(a,b)>0  a<0∧b<0⇒g(a,b)<0

再び新しいgの四条件からg(0,b)=g(a,0)=0を満たすので、
gも、定数項=0、かつ、aだけの項またはbだけの項はない。
そこで、多項式hを用いてg(a,b)=abh(a,b)と書ける。

このとき、gの四条件から、
 a>0∧b>0⇒h(a,b)>0  a>0∧b<0⇒g(a,b)<0
 a<0∧b>0⇒g(a,b)<0  a<0∧b<0⇒g(a,b)<0

これは、fの四条件と同じ。つまり、このプロセスが永久に続いてしまうから、このような多項式はない。
>>629の下から三行目及び二行目のg(a,b)は、h(a,b)に訂正のこと
>>627
arg(αβ)=argα+argβ+2nπ (n∈N)
632132人目の素数さん:03/08/14 19:01
>>631
知ったばかりの工房やからわかりませんわー。すいません
>>632
複素数の基本性質。
>>632
631 は積で arg がどのようになるかを書いてくれてる。
x = r ( cos a + i sin a ) , y = r' (cos b + i sin b)
とおいて x y を計算して整理すること。
635132人目の素数さん:03/08/14 19:10
>>627

z=p(cosα+i*sinα) 、w=q(cosβ+i*sinβ) (p,qは実数,0<p,0<q)
とすると
arg(z)=α+2mπ、arg(w)=β+2nπ (n,mは整数)
zw=pq{cos(α+β)+i*sin(α+β)}  
arg(zw)=α+β+2m'π=arg(z)+arg(w)+2m'π (m'は整数)
z/w=(p/q)*{cos(α-β)+i*sin(α-β)}
arg(z/w)=α-β+2n'π=arg(z)-arg(w)+2n'π (n'は整数)
などなど ・・・ 
636132人目の素数さん:03/08/14 19:17
だめだ・・・もうちょっと勉強してきます・・
z=r(cosα+i*sinα)ってのが 1/z=1/r(cosα-i*sinα)になることはがんばってわかったんですが
何故角度がわざわざ−になるのかわからない・・
637132人目の素数さん:03/08/14 19:31
>>629
なるほど、スッキリしました。
できそうでできなかったので非常に気持ち悪かったのです。
しかも、ついでにこの証明を逆利用してもっと別のケースの悩ましい関数が作れそうです。
数学板に書き込んだ甲斐がありました。
638132人目の素数さん:03/08/14 19:43
>>636
>z=r(cosα+i*sinα)ってのが 1/z=1/r(cosα-i*sinα)になることはがんばってわかったんですが

違う! (表記が間違っているだけかな?)

1/z=1/{r(cosα+i*sinα)}=(1/r)*1/(cosα+i*sinα)
分母の実数化を実行
1/(cosα+i*sinα)=(cosα-i*sinα)/{(cosα+i*sinα)(cosα-i*sinα)}=(cosα-i*sinα)/(cos^2 α+sin^2 α) 
=cosα-i*sinα=cos(-α)+i*sin(-α)
∴ 1/z=(1/r)*{cos(-α)+i*sin(-α)}
つまり
arg(1/z)=-α+2nπ
方向微分係数とはなんですか具体的に教えてください
教科書嫁と言っときます
641636:03/08/14 20:20
いや、これcos(-α)+i*sin(-α)で(cosα-i*sinα)になるんすよ
それはわかるんです。角度が−なるのも、でも何故なるのかが知りたい。
ナンセンスな質問ですかね
642132人目の素数さん:03/08/14 20:41
>>641
何を言っているのか判らん
xの2次方程式
   x^2 −2ax +2a^2 +2a −1 = 0
について,実数解xのとりうる値の範囲を求めよ.
をお教えください.
>>643
x-a=bとおけば
与式⇔b^2+(a-1)^2=2
この円のなかでx=a+bのとりうる範囲をしらべる。つまり傾き-1の直線のなかで
この円とぶつかるもののb切片のとりうる範囲をしらべる。
>644
レスありがとうございます.−3≦x≦1となるようですね.その解法は
素直に理解できるものですね.
ところで,aの2次方程式とみなして判別式D≧0から,xの値の範囲を
求める解法があるようなのですが,理屈がわかりません.お教え頂け
ないでしょうか.
>>645
xの方程式 x^2 - 2ax + 2a^2 + 2a -1 = 0
が実数解を持つような実数aの範囲を求めよ

だったらどうやる?また、

aの方程式 2a^2 - 2ax + 2a + x^2 -1 = 0
が実数解を持つような実数xの範囲を求めよ

だったらどうやる?
まあ言いたいことは、「xとaは対等」だということだ。
1辺がaの正四面体の体積を出す公式があったような気がするのですが、あったら教えてください。
>>647
(√2)a^3/12
649647:03/08/15 10:51
>>648
ありがとうございました。
>>641
角度とか面積というものが、定義などなく天然、自然にあるもの
だという錯覚が原因なのだと思います。角度とはどう定義するのか
を考え、マイナスの数を考えたときのように、それを拡張すること
を考えてはいかがですか?
651132人目の素数さん:03/08/15 14:15
わからないので教えてください。
 次のような実数x,yについての二つの条件
 x^2+2xy-4x+1>0・・・@
 x^2+2xy-3y>0・・・A
に関して、以下の問いに答えよ
(1)「すべてのxに対して@またはすべてのxに対してA」が成立するためのyの条件を求めよ
(2)「すべてのxに対して@またはA」が成立するためのyの条件を求めよ
(3)「すべてのyに対して@またはすべてのyに対してA」が成立するためのxの条件を求めよ
(4)「すべてのyに対して@またはA」が成立するためのxの条件を求めよ
解答しかないので、誰か明快な解説お願いします!


652132人目の素数さん:03/08/15 14:38
>>651
で、何がワカランのよ。
653132人目の素数さん:03/08/15 14:49
それぞれの問題の違いはわかるのですが、どうやって解いていいかわからないのです
>>651
x^2+2(y-2)x+1>0
(x+y-2)^2-(y-2)^2+1>0
-(y-2)^2+1>0
y^2-4y+3<0
(y-1)(y-3)<0
1<y<3

x^2+2xy-3y>0
(x+y)^2-y^2-3y>0
y^2+3y<0
y(y+3)<0
-3<y<0
A が m * m 行列のとき、
A が正則であるときの必要十分条件は、A の固有値が 0 でないことである
ことを証明せよ。

固有値の導出方法はわかるんですけど、こういう証明となるとサッパリです・・。
656トップエリート街道さん ◆BIG6e4aEMg :03/08/15 16:00
>>655
Aが固有値0をもたない、だね。
で、0が固有値であるとはどういう事か定義を書いてみて。
657132人目の素数さん:03/08/15 16:06
>>655
0 でないベクトルが 0 につぶれないから正則。
>646
対等ですか…深い言葉ですね.よく考えてみます.ありがとうございました.
>>656
λが A の固有値であるとき、 (A - λE)x = O (E:単位行列,O:零行列)
において自明でない解 x を持つんですよね。
λ = 0 のとき、 Ax = O で、 x は自明なので A = O
よって A は正則ではない。ということでしょうか。

>>657
0 につぶれないっていうのは非自明ってことですか?
660132人目の素数さん:03/08/15 17:05
>>659
おまえ、頭大丈夫?
>>660
は?ナニ、この人?きっも〜い
>λ = 0 のとき、 Ax = O で、 x は自明なので A = O
>よって A は正則ではない。ということでしょうか。
 
ここおかしい。Ax = O で、 x は自明でないので A は正則ではない。
それと証明を書くときは仮定をキチンと書くところからはじめる。
この文なら“0が固有値であると仮定する。”からはじめないと。
>>662
>ここおかしい。Ax = O で、 x は自明でないので A は正則ではない。
間違えてました・・。指摘ありがとうございます。
仮定をキチンと書いてやってみます。
664山崎 渉:03/08/15 17:50
    (⌒V⌒)
   │ ^ ^ │<これからも僕を応援して下さいね(^^)。
  ⊂|    |つ
   (_)(_)                      山崎パン
665132人目の素数さん:03/08/15 19:19
緊急あげ
急性はげ
比例定数の求め方忘れた
668132人目の素数さん:03/08/16 16:57
fが周期一の周期関数すなわちf(x+1)=f(x)、aが無理数のとき、
 lim{n→∞}(1/n)煤ok=1〜n}f(na)=∫{0〜1}f(x)dx
が成り立ち、これをWeylの一様分布定理ということを知りました。

この証明を教えていただけませんか?
または、証明の出ているホームページでも構いません。
ご存知の方、よろしくお願いします。m(_ _)m
66969マン:03/08/16 16:58
  _n
 ( l    _、_
  \ \ ( <_,` )
   ヽ___ ̄ ̄  )   ろっぴゃくシックスナイン!! 
     /    /
670132人目の素数さん:03/08/16 17:25
>>668
Weyl, H. "Über die Gleichverteilung von Zahlen mod. Eins."
Math. Ann. 77, 313-352, 1916. Reprinted in Gesammelte
Abhandlungen, Band I. Berline: Springer-Verlag, pp. 563-599,
1968. Also reprinted in Selecta Hermann Weyl. Basel,
Switzerland: Birkhäuser, pp. 111-147, 1956.
671質問です:03/08/16 18:48
コインを投げて n 回連続で表が出る確率って 1/2^n ですよね?
ここまではわかるんですが、
x 回の試行中に n 回連続で表が出る確率っていうのは
どう求められるんでしょうか。
たとえば100回投げた中で10回連続表が出る確率とか。
100回より200回、200回より300回のほうが
10連続表の可能性は高まるわけですよね?
マルチウゼー
>>671
理論的には、二項分布で計算できるが、面倒臭いだけ。
少なくとも、学校の試験に出る問題ではない。
一体何のためにこんな問題聞くの?
674132人目の素数さん:03/08/16 19:16
673 名前:ぼるじょあ ◆yEbBEcuFOU 投稿日:03/08/16 18:54
>>671
理論的には、二項分布で計算できるが、面倒臭いだけ。
少なくとも、学校の試験に出る問題ではない。
一体何のためにこんな問題聞くの?
675671:03/08/16 20:37
>673
何のためかっていうと、くだらないんですが、
賭事で勝つまで賭金を倍にしてくってやりかた(マーティンゲイル)が
どの程度有効なのか、ちょっと興味がありまして。

一単位賭けて負けたら二単位賭ける、二単位でだめなら四単位――
これを実践していくと最後に一回勝った時点でつねに一単位分黒字になるわけですが、
たとえば3連敗するとその3回分の負けの総額は1+2+4=7単位分の負けになります。
つねに一単位ずつ勝つことを目標にするとして、
3連敗するまでに7単位稼げる(7勝する)可能性はどの程度のものか。
そんなふうなことに興味があります。

面倒臭いってことなので、もうちょっと自分で勉強してみます。レスthxでした。
とりあえず確率 p で起こる事象が、n 回連続で初めて起こるまでに
試行する回数の平均は 1/p + 1/p^2 + ... + 1/p^n だ。
誰かこの問題今日中に解いてくれ。
5時間考えたが無理だったよ。

↓の数列の初項から第n項までの和を求めよ。
1, 1+2, 1+2+3, 1+2+3+4, ・・・・・・+n, ・・・・

この問題が解けないと、気になって夜も眠れないだろうから頼むよ。
>>677
普通にΣ二回使えば計算できるだろ。
679677:03/08/16 21:23
>>678
やってくださいよ。
この問題は例題が付いてないんだ。
実数を成分とする正方行列A,Bがいずれも逆行列をもたず、A+Bは逆行列をもち、
さらに、AB=BAならば、AB=O(零行列)であることを証明せよ。
京大プレの問題ですが解答マダもらってません。 おねがいします。

質問スレに書いてもウダウダ言うだけでしたのでお願いします。
681132人目の素数さん:03/08/16 21:25
(1)a,b,cは整数とする。xに関する3次方程式x^3+ax^2+bx+c=0が有理数の解をもつならば、その解は整数であることを示せ。
(2)方程式x~3+2x~2+2=0は、有理数の解をもたないことを示せ。


>>680-681
マルチ
>>677>>679
この数列の第k項目をa(k)とすると、
 a(k)=k(k+1)/2
∴煤ok=1〜n}a(n)=煤ok=1〜n}k^2/2+煤ok=1〜n}k/2=n(n+1)(2n+1)/12+n(n+1)/4
 =n(n+1)/12×(2n+1+3)=n(n+1)(n+2)/6
>>679
例題?ヴァカか?それは基本問題だぞ?
685132人目の素数さん:03/08/16 21:37
>>682
しね
686 ◆BhMath2chk :03/08/16 21:40
>>680
Aを対角成分が(1,1,0)の対角行列で
Bを対角成分が(0,1,1)の対角行列とすると
A,Bは逆行列を持たずA+Bは逆行列を持ちAB=BA≠0。
687132人目の素数さん:03/08/16 21:43
>>686
馬鹿発見!!
688132人目の素数さん:03/08/16 21:43
687 名前:132人目の素数さん :03/08/16(土) 21:43
>>686
馬鹿発見!!
689132人目の素数さん:03/08/16 21:44
馬鹿は>>687
690132人目の素数さん:03/08/16 21:44
688 名前:132人目の素数さん :03/08/16 21:43
687 名前:132人目の素数さん :03/08/16(土) 21:43
>>686
馬鹿発見!!

>>686
正方行列
692132人目の素数さん:03/08/16 21:47
691 :132人目の素数さん :03/08/16 21:46
>>686
正方行列
693132人目の素数さん:03/08/16 21:47
>>689
自分の論証に自信持ちすぎw
694132人目の素数さん:03/08/16 21:47
馬鹿は>>688,689でした(プププププ
695132人目の素数さん:03/08/16 21:47
>>694
2ch初心者ですか?馬鹿はYahoo!に帰って下さい。
697132人目の素数さん:03/08/16 22:01
>>696
でけっきょく解けます?
698132人目の素数さん:03/08/16 22:04
>>697
2次の正方行列だよね。簡単な問題だよ。
ネタバレとわかってうpするわけないけどね。
>>686
いや〜。小気味良い回答だねぇ。
700get
701132人目の素数さん:03/08/16 22:19
>>698 俺はもう受け終わったうえで解答漏れ防止のため解答貰えなかったんですが
702132人目の素数さん:03/08/16 22:20
結局解けないんだろ(プ
>>701
もう寝るから明日解答をアップするよ
704680:03/08/16 22:34
>>698
言い訳はいいから、さっさと解答書いて下さい。
それとも、解けないんで負け惜しみですか?
夏ですな
ですな
707132人目の素数さん:03/08/16 22:41
>>704
煽り方が下手すぎw
708質問:03/08/16 22:48
ベクトルの(1-t):tとかs:(1-s)とかがわかりません。
709数学野郎 ◆eNwncubcDk :03/08/16 22:49
不等式の解き方について質問です

2/3(4^n-1)=1000から4^n>1051になる途中式が
わからないので手の空いている先生いらっしゃいましたら
ご教示ください。
>>709
ならない。
>>708
あっそ。
712132人目の素数さん:03/08/16 22:54
4^n-1≧1000・3/2
4^n-1≧1500
4^n≧1501
>>712
2/3(4^n-1)=1000 → 4^n-1≧1000・3/2 とでも言うのか?
714数学野郎 ◆eNwncubcDk :03/08/16 22:59
>>710
教えてくださってありがとうございます、数列の問題なんですけれど

問題文
初項が2、公比が4の等比数列の初項から第n項までの和が初めて1000を
超えるときのnの値を求めよ

模範解答

S=2(4^n-1)/4-1
=2(4^n-1)/3

S>1000とすると2/3(4^n-1)>1000
ゆえに4^n>1501
この不等式を満たす最小の自然数nはn=6

なんですけれどこの問題での709で書いた事項です、
言葉足らずで申し訳ありません。
>>713
それはいいんだが、
4^n≧1501 ⇒ 4^n>1051
ここが・・・
>>714
両辺2/3倍して-1を移項するだけ。
訂正
×2/3倍
○3/2倍
718数学野郎 ◆eNwncubcDk :03/08/16 23:05
先生ありがとうございました。その方法でやってみます、助かりました。
719数学野郎 ◆eNwncubcDk :03/08/17 01:04
今日2つめの質問なんですけれど、
http://rx.sakura.ne.jp/~yuhgi/high/img-box/img20030817005530.jpg
リンク先が問題と模範解答です。(三角関数)

質問点はリンク先の波線部分なんですけれど3-√5/6=6-2√5/6*2=(√5-1)^2/6*2
ですけれども分母・分子に2を掛ける理由とその次に分子の6-2√5を(√5-1)^2の形に変形する
理由を手の空いている先生いらっしゃいましたら教えてください。
>>719
sinα = √(3-√5/6)
これは二重根号の形だから√内を (√a+√b)^2 = (a+b)+2√(ab) の形に直す作業が必要。
それをあらかじめやっておいたのが波線部。
721数学野郎 ◆eNwncubcDk :03/08/17 01:09
>>720
夜分遅く教えてくださってありがとうございました。
722132人目の素数さん:03/08/17 02:28
>>719
見れねぇよ
んなことぁない
724数学野郎 ◆eNwncubcDk :03/08/17 02:45
>>722
ごめんなさい。
725数学野郎 ◆eNwncubcDk :03/08/17 02:47
0°<α<45°、0°<β<45°でsin2α=2/3、sin2β=4/3であるとき
sina=□、tanβ=□である。

というのが719のリンク先の問題です。
726132人目の素数さん:03/08/17 06:36
a^i=?
2ちゃんねらーなどの間で、人気のある寺院が有ります。
これらの寺院にて諸願成就を祈願しましょう。(爆)

(1-4件目)  http://www.z-shoten.or.jp/
http://www.tctv.ne.jp/matuti/
http://www5b.biglobe.ne.jp/~ryumyoin/
http://www1.ocn.ne.jp/~tatsueji/
(5件目)
寺院名    吉祥山唐泉寺
通称     江戸川不動尊
所属宗派  真言宗泉涌寺派
住所     〒133−0051
        東京都江戸川区北小岩七丁目10−10
        京成電鉄の小岩駅から徒歩約15分
        (JR線の小岩駅は、
        京成電鉄の小岩駅とはもの凄く離れているので不可。)
電話番号  03−3658−4192
住職     高田正圓
        (女住職で、先代住職(高田真快)の奥さんであった模様。) 
本尊     不動明王
祈祷日及び祈祷時刻
        通常は毎日午前6時より(150分前後かかる模様)
        行われるが、毎月28日には(不動明王縁日として)
        午前11時にも(2時間ほどかかる模様)行われる。
祈祷料
(普通護摩)   3000円 5000円 10000円
(特別護摩)   30000円(21日間) 100000円(108日間)
          300000円(365日間)
729132人目の素数さん:03/08/17 08:32
>>725
2倍角の公式→二乗→sinだけの式にする→複二次方程式→解の平方→条件に当てはまる方
730668:03/08/17 10:16
>>668どなたかお願いできせんですか?

>>670さん、独逸語の原論文は読めませんし、手に入れることができません。
731132人目の素数さん:03/08/17 10:31
>>730

これ、式あってます?

>>668
>lim{n→∞}(1/n)煤ok=1〜n}f(na)=∫{0〜1}f(x)dx
732668:03/08/17 10:57
>>731
間違ってますか?もしそうなら済みませんでした。
733668:03/08/17 11:22
>>731
今、確認しましたが、式はこれで正しそうです。
 lim{n→∞}(1/n)煤ok=1〜n}f(na)=∫{0〜1}f(x)dx
734132人目の素数さん:03/08/17 12:35
>>733
(1/n)納k=1〜n]f(na) = (1/n)*f(na)納k=1〜n] = (1/n)*f(na)*n = f(na)

??
735668:03/08/17 12:38
>>733
ああ、そういうことか。済みませんでした。修正します。
× lim{n→∞}(1/n)煤ok=1〜n}f(na)=∫{0〜1}f(x)dx
○ lim{n→∞}(1/n)煤ok=1〜n}f(ka)=∫{0〜1}f(x)dx
736132人目の素数さん:03/08/17 12:39
n/12,360/nがともに整数になるような自然数の数を求めよ。
すみません。どうしても分からないです。
素因数分解をして、次は
>>736
n/12が整数だから、nは12の倍数。
360/nが整数だから、nは360=12*5*3*2の約数。
∴ n=12,12*2,12*3,12*5,12*2*3,12*2*5,12*3*5,12*2*3*5
   =12,24,36,60,72,120,180,360
自然数の数
個数?
360/12=30
30の約数を数える
X^2+3*X*Y+2*Y^2-X-2
因数分解せよ

どうしても解けませんお願いします。
(x+y+1)(x+2y-2)
>>741
たすきがけを使えば解けますが…
 X^2+3XY+2Y^2−X−2=X^2+(3Y−1)X+2(Y+1)(Y−1)
 ={X+2(Y−1)}{X+(Y+1)}=(X+2Y−2)(X+Y+1)
ありがとう御座いました。

勉強しなおします。
745132人目の素数さん:03/08/17 13:39
記述を意識した完璧な解答作ってみて貰えませんか?
ただし、グラフの概念は用いずに。

xの範囲を求めよ。
@ √(2x+5)>x+1
A √(5-2x)<2x+1
yada
吊り⇒>>745は無視で
>>745
1)-5/2≦x<2
2)1/2<x≦5/2
749132人目の素数さん:03/08/17 13:54
>>748は釣り師
違うって
751745:03/08/17 14:00
>>748
ありがとうございます。
もっと厳密な証明をお願いします。
隊長、釣り師が発生しますた!!
753______:03/08/17 14:20
2ちゃんねらーなどの間で、人気のある寺院が有ります。
これらの寺院にて諸願成就を祈願しましょう。(爆)

(1-4件目)  http://www.z-shoten.or.jp/
http://www.tctv.ne.jp/matuti/
http://www5b.biglobe.ne.jp/~ryumyoin/
http://www1.ocn.ne.jp/~tatsueji/
(5件目)
寺院名    吉祥山唐泉寺
通称     江戸川不動尊
所属宗派  真言宗泉涌寺派
住所     〒133−0051
        東京都江戸川区北小岩七丁目10−10
        京成電鉄の小岩駅から徒歩約15分
        (JR線の小岩駅は、
        京成電鉄の小岩駅とはもの凄く離れているので不可。)
電話番号  03−3658−4192
住職     高田正圓
        (女住職で、先代住職(高田真快)の奥さんであった模様。) 
本尊     不動明王
祈祷日及び祈祷時刻
        通常は毎日午前6時より(150分前後かかる模様)
        行われるが、毎月28日には(不動明王縁日として)
        午前11時にも(2時間ほどかかる模様)行われる。
祈祷料
(普通護摩)   3000円 5000円 10000円
(特別護摩)   30000円(21日間) 100000円(108日間)
          300000円(365日間)
隊長、釣レ)師退散σ祈祷をUませぅ!!
755数学野郎 ◆eNwncubcDk :03/08/17 17:37
漸化式についての質問です。
http://rx.sakura.ne.jp/~yuhgi/high/img-box/img20030817173352.jpg
問題と質問点はリンク先に書きました。手の空いている先生、教えて下さる先生
いましたらご教示よろしくお願いします。
>>755
公式っつーか等比数列の和
逆から書いてみるとわかるかな?
>>755
r^n-1=(r-1)(r^(n-1)+r^(n-2)+・・・+1) ←因数分解
両辺をr-1で割って以下略

公式としては
1+r+r^2+・・・r^n=(r^(n+1)-1)/(r-1)
推測したら証明しなきゃだめよん
あと、1行目微妙に間違ってるよ
760数学野郎 ◆eNwncubcDk :03/08/17 18:37
756先生、757先生、758先生、759先生教えてくださってありがとうございます。

>>758
わかりました。
761132人目の素数さん:03/08/17 19:35
762132人目の素数さん:03/08/17 19:45
>>668
[1] H. Weyl: Über die Gleichverteilung von Zahlen mod Eins. Math. Ann. 77 (1916), 313 - 352.
[2] T. M. Apostol: Introduction to Analytic Number Theory. Berlin, Heidelberg, New York 1976.
[3] Hua Loo Veng: Introduction to Number Theory. Springer - Verlag, Berlin, Heidelberg, New York,1982.
証明そんなにむずかしくないんでわ?
>>668
ってステートメントすこしおかしくないか?α無理数をとって関数fを
f(x)=1 (x=mx+n m,n:整数 とかけるとき) 0 (そうでないとき)と定義すると
ルベーグ可測関数で周期関数だけど>>668の左辺=0、>>668の右辺=1になる。
成立しないんだから証明しようがない。fが連続ならf(x)=exp(inπx)で一様近似して
やれば簡単に示せるけど。
f(x)=1 (x=mα+n m,n:整数 とかけるとき) 0 (そうでないとき)と定義すると
この同じαについて>>668の左辺=0、>>668の右辺=1になる。
に訂正。
766132人目の素数さん:03/08/17 20:48
質問です

こないだテレビでこんなことを放送していました。

アメリカでギャンブルで億万長者になった男がいる。そのギャンブル必勝法
とは
ルーレットの黒赤のみの2分の1で勝負。
黒が5回連続したら、次赤に全部かける。逆に赤が5回連続したら、次全部
黒にかける。
6回連続同じ色がくる確立は1/2*1/2*1/2*1/2*1/2=1/32だからだ、反対の
色をかけるほうが確立が高いという。
疑問なのが前にどんな色が連続していおうと、結局黒と赤がくる可能性
は1/2なんじゃないでしょうか?まちがっていたら教えてください。
次の式を簡単にしなさい
1 √6+√2/√6−√2+√6−√2/√6+√2
2 1/1−√2+√3
3 (√2−√6+√12)^2

この問題ですがあんまりよくわからないんですよ。答えは
1 4
2 √6−√2+2/4
3 33−18√2
ってわかるんですけど解き方よくわかりません。よろしくお願いします
768132人目の素数さん:03/08/17 20:52
>>767

問題間違っている。正確に写しなさい。
769sage:03/08/17 20:53
まるち
>>767
聞く前に、もう一度>>1を熟読してください。
>>767
( )をうまく使ってね

例.
1+√2/3≠(1+√2)/3
スマソ・・例えばこんな感じですか?誤りあったらお願いします

(√6+√2)/√6−√2+(√6−√2)/√6+√2
1/(1−√2+√3)

あとマルチとか言われているけど、このスレにしか書いてないので、まぁ信じる信じないは勝手ですが
>>766
その質問には、数学は答えられない。
ルーレットの物理的特性とか、変な仕掛けがしてある可能性とか、
そういうことを考え出したらきりがないから、数学では

「赤黒どちらも、過去の結果と関係なく、
1/2の確率で出るものと決めましょう」
               ~~~~~~~~~~~~
として議論を始めるのだ。「決まっている」のではない。
我々がそう考えることに「決める」のだ。
>>772
>あとマルチとか言われているけど、このスレにしか書いてないので、まぁ信じる信じないは勝手ですが
=http://science.2ch.net/test/read.cgi/math/1060617593/605
775772:03/08/17 21:51
>>774
そのスレに書いている香具師は偽者です。証明できるものは何もありませんが・・
IDないですからねこの板。これ以上書くとスレ違い&スレ汚しになるので退散します。すみませんでした。
776132人目の素数さん:03/08/18 13:13
>>772
分母の有理化をはかります。

(√6+√2)/(√6−√2) + (√6−√2)/(√6+√2) = (√6+√2)^2/{(√6−√2)(√6+√2)} + (√6−√2)^2/{(√6+√2)(√6−√2)}
=(8 + 4√3)/4 + (8 - 4√3)/4 = 4
この問題は通分しても出来ます。
(√6+√2)/(√6−√2)+(√6−√2)/(√6+√2) = {(√6+√2)^2 + (√6−√2)^2}/{(√6+√2)(√6−√2)} = 16/4 = 4

1/(1−√2+√3) = {(1-√2) - √3}/[{(1-√2) + √3}{(1-√2) - √3}] = {(1-√2) - √3}/{(1-√2)^2 - 3} = {(1-√2) - √3}/(-2√2)
= -{(1-√2) - √3}*√2/(2√2*√2) = -(√2 - 2 - √6)/4 = (√6 + 2 - √2)/4

nまでの和は(x(x+1)/2で表せることの証明ってどうやるんでしょうか?
いざやってみようかと思ったのですがわかりません。
どなたかよろしくお願いします。
778132人目の素数さん:03/08/18 14:22
>>777
nまでの何の和ですか?
779132人目の素数さん:03/08/18 14:34
ここに200人の人間が一列に並んでいる。
それらには一番前から順に1、2、3、4、5、・・・200の背番号が降られている。

それらに、ある正の整数を見せた。
すると
1「それは私の番号で割り切れますね」
2「それは私の番号で割り切れますね」
3「それは私の番号で割り切れますね」
・・・
200「それは私の番号で割り切れますね」

全員「それは私の番号で割り切れますね」と答えた。
しかし、実際には隣り合った二人が嘘をついたという。

嘘をついたのは何番か、また、全員に見せた数はなにか。
>>778
1からnまでの合計です。
舌足らずですいません。。
ゆるさん!
謝罪しる!
782132人目の素数さん:03/08/18 15:49
>>780
舌足らず?
言葉足らずでは?


と揚げ足とって見るテスト


ついでに言うと>>779の問題はひん
>>779
1327927515090260884407345538562367745796828278681721394601759928808007945120777126248000
>>779
127と128が嘘をついた。
785132人目の素数さん:03/08/18 16:14
>>779
無限に存在する
783が嘘をついた
>>784にイピョー
おれも>>783>>784と同じになった。
require 'Mathn'

l=Integer(1)

for i in 1..200 do
l=l.lcm(i)
end
puts l/(127*2)

結果

1327927515090260884407345538562367745796828278681721394601759928808007945120777126248000
199と200でしょ?

嘘をついたってことは割り切れないわけで
合成数ではないわけで
素数なわけで
>>789

200なわけない。8と25がほんとなら200の倍数になるので。
ウソをいってる香具師は素数べきであることが必要。
791132人目の素数さん:03/08/18 16:38
>>789
200=8*25
で、ある数は8でも25でも割り切れるのだから200でも割り切れる
792791:03/08/18 16:39
かぶった・・・
ごめん
ねばまいん
794785:03/08/18 16:40
私は間違っているのですか
全員に見せた数は一つには定まらないと思うのですが
無限に存在するのはその通りでつよ。
具体的に一個上げるとすれば、>>788てこと。
>>794
あってんでわ?すくなくとも
>>783の整数×(201以上の素数)
はすべて題意をみたすから。ほかにもあるし。ウソをいってる2人は特定できるけど見せた数字は
特定できないね。
797785:03/08/18 16:48
>>795
>>796
ありがとうございます
798132人目の素数さん:03/08/18 17:05
VをC上の{0}でない有限次元ベクトル空間とし、
線形変換f:V→Vが条件
Ker f = Im fを満たしているとする。
ただし、
Ker f = {vV|f(v) = 0}
Im = {f(v)|vV}
である。

(1)Ker f != {0} と Im f != Vを示せ。

(2){v1, . . . , vn} をIm f の基底とする.
f(wi) = vi (i = 1, . . . , n) を満たすVの
ベクトルw1, . . ., wn をとると,
{v1, . . . , vn, w1, . . ., wn} は1 次独立で
あることを示せ.

(3) (2) における{v1, . . . , vn, w1, . . ., wn}
はV の基底であることを示せ.

(4) V の基底{v1, w1, v2, w2, . . . , vn, wn}
に関するf の行列表示を求めよ.
f!ってなんだろ?
(σ・∀・)σゲッツ!! 800
!=で≠を表すらしい
802132人目の素数さん:03/08/18 17:20
>>777

 S = 1 + 2 + 3 + ・・・・ + (n-2) + (n-1) + n
とおき、この和の順を逆にして表してみる。
 S = n + (n-1) + (n-2) + ・・・・+ 3 + 2 + 1
これらを上下に並べて書き、順に上下の対応する数を足し合わせてみる。

   S = 1   +  2  +  3  + ・・・・+ (n-2) + (n-1) + n
+) S = n   + (n-1) + (n-2) + ・・・・+  3  +  2  + 1
 ̄ ̄ 2S = (n+1) + (n+1) + (n+1) + ・・・・+ (n+1) + (n+1) + (n+1) ̄ ̄ ̄

のように足し算するべき各数は n個の全て同じ n+1 になった。これで和が簡単に計算できる。

 ∴ 2S = n(n+1)

よって S = n(n+1)/2 となる。

他の等差数列の和もこの様な計算で求まる。試してみるとよいでしょう。
>>801
なるへそ。
804798:03/08/18 17:21
そうです。どうしても職業柄、プログラミングっぽく
書かいちゃうくせがあって・・・。
どなたかもう答え分かった方いらっしゃいますか?
>>804
(1)は完全列0→Kerf→V→Imf→0に着目してdimKerf+dimImf=dimVであることを利用する。
(2)は普通に蚤ivi+巴iwi=0と仮定するとこからはじめる。これにfを作用させて
どこにうつるか考える。
(3)は(1)と(2)のくみあわせ。
(4)は定義のまんま。
>(3)は(1)と(2)のくみあわせ。
これは(2)とdimKerf+dimImf=dimVを組み合わせるといういみでふ。
>>802
ありがとうございました。
>>766
それ見た。あれはおそらくディーラーを買収してる。
809独り言:03/08/18 20:29
赤赤赤赤赤赤 1/32
赤赤赤赤赤黒 1/32
>>808-809

766じゃないけど、やっぱり黒に賭けたほうがいいんじゃないのか?
経験的には6回同じ色が続いたら、変えたほうがいいってのは定説だな。
確率的にはどっちの色がくる確立も同じなんだけどね。
連続して同じ色が出るのはルーレットの器械が偏ってるから、と考えられるので
次も同じ色の可能性がわずかに高い。
上手いディーラーだと狙った数にピンポイントで球をのせることが出来る。
よって賭けてない色の方が確率がわずかに高くなると思う。

>>809
0と00を無視していますね。
普通のカジノは盤を回すのと玉を投げるのは別の人間がやりますが何か?
815132人目の素数さん:03/08/19 10:12
「円周上に9個の点がある。次の問いに答えよ
1)対角線は何本引けるか。
2)三角形はいくつできるか。」

という問題がさっぱりわからないのですが、どのような計算をすればよろしいでしょうか。お願いします!!
816132人目の素数さん:03/08/19 11:19
>>815
その設問なら
1)いったい何の対角線なのか?
2)無数にできる。
となります。何を問われているのか私にもさっぱり判りません。

問題は正確に。
次の式を簡単にせよ

1) √45-√20+√125
2) √3(√12+√6)
3) (1/√3)-(1/√12)-(1/√48)
4) (2√2)/(√6-√2)
5) √18


書き方あってるかな…
おねがいします(;´Д`)ヤリカタワスレチマターヨ
818132人目の素数さん:03/08/19 12:06
(1)3√5-2√5+5√5=6√5

これで思い出せ
819132人目の素数さん:03/08/19 12:19
名前なんだったかな?
分数の極限値が0か∞か分からないとき、
分母、分子をとにかく微分するってやつ。
大学受験のとき、タブーと言われながらも、こっそり使ってたアレ。
名前わすれたんだけど、何の定理だったっけ?
820132人目の素数さん:03/08/19 12:25
思い出した。ロピタルの定理だ。
>>816
いぢわるはやめて9角形と思ってあげようよ
822132人目の素数さん:03/08/19 12:54
>>821
わかった!貴方に免じてここは一つ大人になろう。

>>815
円周上の異なる9個の点で作られる九角形について
1) その対角線は、9頂点から隣り合わない2点を結べば1つの対角線ができるので
 隣り合ってもよいとして2点を選ぶ仕方は C(9,2)=9!/{2!*(9-2)!}=9*8/2=36 通り
 そのうち隣り合っている2点は 9通り あるから、求める対角線の数は 36-9=27 通り
2) 9頂点の何れかを頂点とする三角形は、9頂点から3点を選べば1つの三角形ができるので、
 その三角形の数は C(9,3)=9!/{3!*(9-3)!}=9*8*7/(3*2)=84 通り
>>818サンクス
でもそれだと2)3)4)の解き方が解らない
解き方同じなんですか?

あと5)の答えは 3√2 で正解?
824あっぺ:03/08/19 13:37
>>823
(5) 正解

(2)はカッコを展開すれば(1)と同じになる
(3)分母の有利化をして通分すれば(1)と同じになる
分母の有利化:1/√3=(1×√3)/(√3×√3)=√3/3
(4)分子、分母に(√6+√2) をかけると(1)と同じになる
825194:03/08/19 15:01
(3/10+(2/5)i)^nでnを∞まで飛ばす問題なのですが、
極形式で表せません。Oに収束かな?と思ったのですが。
826132人目の素数さん:03/08/19 15:08
>>825
何でやねん?
√{(3/10)^2+(2/5)^2}=1/2
3/10+(2/5)i=1/2*(3/5+4/5*i)
3/5=cosθ、4/5=sinθ (0≦θ<2π) とおけて ・・・ 
827194:03/08/19 15:15
>>826さん
あれ?絶対値を出す時って、いきなり二乗の和をとってわらないとだめなのですか?
まず(3/10)(1+2i)とくくって、それから、1+2i部分の二乗の和をとって
それでわってみたのですが。
828132人目の素数さん:03/08/19 15:33
>>827
たとえば a+b*i=r(cosθ+i*sinθ) とするやん
すると |a+b*i|=r
一方 |a+b*i|^2=(a+b*i)(a-b*i)=a^2-(b*i)^2=a^2-(-b^2)=a^2+b^2 やん
つまり r=√(a^2+b^2) やんか な。
>nを∞まで飛ばす

陽気な表現だ
830194:03/08/19 15:52
>>828ごめんなさい。
ふつうに係数括り間違いしてました。しょっぱなから。

>>829
私のかかわってた先生はみんなそういってましたよ。
831815:03/08/19 19:14
ご教授いただきましてどうもありがとうございました!
ちなみに、問題をそのまま丸写しにしたので、責めるなら問題を作った教師を責めてください(ワラ
ttp://toricco.s25.xrea.com:8080/ca/file/ca086.jpg
832132人目の素数さん:03/08/19 19:20
対角線は27本

できる三角形は、場合わけが必要だな
833132人目の素数さん:03/08/19 19:47
潮吹きっ子のあちゃんの感度抜群のオマンコがいいですよ。
男優さんに指姦されて大量の潮吹き!
その後のフェラシーンでは亀頭をチロチロ高速回転フェラで翻弄し,本番シーンへと突入します。
小さめのオマンコは締りがよさそうで男優さんもタジタジ!
形のいいバストをモミモミしながら果てていくのでした。
現役AV女優から懐しのAV女優まで
さらにセーラー服ものまで全てモロ!!
無料ムービー観て!
http://www.ncdonald.com/
>>824解説サンクス
835132人目の素数さん:03/08/19 22:28
836132人目の素数さん:03/08/19 23:27
スタイネルの定理?ってなんですか・・・
>>836
スタイネル点の存在に関する定理。そのくらい自分で調べろ。
 x=1/(√7+√6),y=(√7-√6) のとき次の式の値を求めよ
(1)x+y
(2)xy
(3)x~2+y~2

解き方、答え方何一つワカンネ
お願いします
有理化
夏休みも終わりに近づいて
今後ますます宿題教えて厨が増えるんだろうな
「≒」ってなんて読むのですか?
文系大学生ですが家族友人だれも答えられませんでした
>>838
x=yだからx+y=2y,xy=y^2,x^2+y^2=2y^2。
843132人目の素数さん:03/08/20 21:30
質問します。

知り合いに「無限大は数ではない」と主張している人がいるのですが、
これは本当でしょうか?

私は∞が数ではなかったら演算出来ないのではないかと考え∞も数だと思っています。。。
また彼は単に∞を何かの記号と勘違いしているのではないかと私は想像しています。

よろしくお願いします


>>841
nearly equalではないでしょうか?
http://ha7.seikyou.ne.jp/home/namingt/call.htm
844132人目の素数さん:03/08/20 21:39
  _、_
( , ノ` )    いいと思うが・・・
  \,;  シュボッ
    (),
    |E|
http://homepage3.nifty.com/manko/
845132人目の素数さん:03/08/20 21:55
>>843
では、∞は数と同様の演算が成り立つのですね。
∞-∞=0 ∞/∞=1 
しかし、n→∞ のとき n^2-n→∞-∞  (n+1)/(n^2-n-2)→∞/∞ だけれど、
n^2-n=n^2(1-1/n)→∞  (n+1)/(n^2-n-2)=(n+1)/{(n+1)(n-2)}=1/(n-2)→0 だよ。
 
>>845
では0は数ではないんですね。
847132人目の素数さん:03/08/20 22:01
>>845
ごめんなさい自然数などと同等の演算が出来ない事は知ってました。
2*∞=∞
など

∞は数には含まれないって事でしょうか?
848132人目の素数さん:03/08/20 22:04
>>846
どうして?
失礼します。

(cos-sin)/o(cos+sin)=(1-tan)/(1+tan)

の証明が分かりません。
(cos-sin)/o(cos+sin)が
-tan+(1)/(tan)
になった辺りで既に止まっています。(これ自体正しいかどうか分かりませんが)
ちなみに問題集の解答は証明略となっていて、ヒントの欠片もありません。
よろしければアドバイスお願いします
850841:03/08/20 22:09
>>843
サンクス。
>>849
(cos-sin)/(cos+sin)=(1-tan)/(1+tan)だったら左辺の分子分母をcosでわりたまへ
852849:03/08/20 22:13
失礼。記入ミスです。

正しくは
(cos-sin)/(cos+sin)=(1-tan)/(1+tan)
で、初めの分数のoはミスです。
853849:03/08/20 22:15
とと、訂正の前に解説サンクス。
早速やってみます。>>851氏に感謝
854132人目の素数さん:03/08/20 22:16
>>852

(cos-sin)/(cos+sin)={(cos-sin)/cos}/{(cos+sin)/cos}=・・・
>>848
0が数と同様の演算が成り立つなら0/0=1。
(2x)/x=2でx−>0として0/0=2。
856132人目の素数さん:03/08/20 22:23
>>855
0は数として △/0 は不能と定義されている。
>>845
∞は数として∞−∞,∞/∞は不能と定義されている。
858220:03/08/20 22:35
>>857
ははは ひっかかった!
∞−∞,∞/∞ は不定だす。
>>847
>彼は単に∞を何かの記号と勘違いしているのではないかと
∞というのは単なる記号です。(果てしなく大きいものを表す時に登場しますね。)
この果てしなく大きいという言葉には数学的に曖昧な部分が多いのですが
大学ではε-δ論法という極限概念を定義するときに"果てしなく大きい"や"無限に小さい"というものを数学的に定義します。
>∞は数には含まれないって事でしょうか?
"数"というものの範囲をどうするかによると思います。少なくとも一般的な定義での"実数"では無いでしょう。
虚数単位の"i"のようなものと思ってよいのでは?
(しかしiは四則演算が定義されていますが、∞には定義されない演算がたくさんありますからこのあたりは大きな違いがありますね。)

結局"数"という言葉には広がりがあるので∞を"数"だと思うかどうかは人ぞれぞれなのでは?
(というより私は"数"の定義によりけりだと思います。)
>>858
不定って何。
不能って何。
220って何。
>>860
◆ わからない問題はここに書いてね 122 ◆
http://science.2ch.net/test/read.cgi/math/1061227058/220-222

862843:03/08/20 23:02
>>859
丁寧な返答ありがとうございました
>>860
不定って、奥さんが他の家の旦那とできてしまうこと。
不能って、旦那が奥さんの不定の原因を作ること。
220って、ヴァカっていうこと。
220の約数1,2,4,5,10,11,20,22,44,55,110,220。
1+2+4+5+10+11+20+22+44+55+110=284。
284の約数1,2,4,71,142,284。
1+2+4+71+142=220。
上底35cm 下底40cm 高さ25cmの綺麗な台形(正台形?)の
角度が出せません、、。  お願いします。
 0≦x≦3 ⇒ (x−a)(x+a−2)≦0
な定数aの値の範囲を求めよって問題ですけど
p(a) ⇒ {0≦x≦3 ⇒ (x−a)(x+a−2)≦0}
な条件p(a)を求めろってことですか?
>>865
台形で、平行でない2辺の長さが等しいものを等脚台形という。
で、鋭角をθとすれば、tanθ=10。あとは関数電卓にお任せ。

>>866
問題文は

全ての実数xに対し「0≦x≦3 ならば (x-a)(x+a-2)≦0」
となるような実数aの範囲を求めよ

という意味でいいのかな?
まあその解釈でも間違ってはいないが、
そんな言い換えをしても何も見えないと思う。
A→(B→C)≡(A∧B)→C
p(a)=(a=10)。
869132人目の素数さん:03/08/21 04:42
>>868
?????
868の指摘は正しい
この問題は必要十分を求めなければ駄目
>>867
>まあその解釈でも間違ってはいないが、
...って、適当なこと言ってんじゃねーよ。
あえて書くなら
>p(a) ⇒ {0≦x≦3 ⇒ (x−a)(x+a−2)≦0}
じゃなくて
p(a) ⇔ {0≦x≦3 ⇒ (x−a)(x+a−2)≦0}
だろうが。

それに対する皮肉が>>868だろ、きっと。

問題文は間違いじゃないが、普通に書けば
「0≦x≦3を満たす全ての実数xに対し(x-a)(x+a-2)≦0となるような
実数aの範囲を求めよ」
だろうよ。
んで、y=f(x)=(x-a)(x+a-2)のグラフは下に凸だから、
「f(0)≦0かつf(3)≦0」⇔「0≦x≦3を満たす全ての実数xに対しf(x)≦0」
>867-872
ありがとうございます。あえてでも書かないと分からないです。
>871
そう書いてありました>問題文
>872
なるほどそうやって解いていくんですね。
874132人目の素数さん:03/08/21 11:34
アルキメデスの公理は∞を実数として認めないんだな、これが
875132人目の素数さん:03/08/21 13:27
aを定数とするとき、√2x^2-2x-√2=0を解け。

の解き方がわかりません。解答はx=(√2±√6)/2になっています。

よろしくお願いします。
>>875
両辺√2で割って解の公式
877875:03/08/21 13:40
>>876
ありがとうございます。
解の公式を使わないやりかたってないですか?
ないならあきらめますが。。
>>877
平方完成でできる
879875:03/08/21 13:48
>>878
できました。ありがとうございました。
881132人目の素数さん:03/08/21 16:17
周囲の長さが24である三角形の辺の長さがすべて整数であるとき、
互いに合同でないものは何個あるか?

この問題がどうしてもわかりません。どなたかお願いいたします。
>>875
>aを定数とするとき、√2x^2-2x-√2=0を解け。

aってどこに使われてるの???
>>882
使われてなくたっていいんだよ

東京を首都とするとき、√2x^2-2x-√2=0を解け。
884132人目の素数さん:03/08/21 17:42
>>881
12個か?
代数を勉強しているのですが、群について剰余類の概念が出てきて
つまづいてます。周りに数学屋さんがいないので、教えていただきたいのですが。

群Gの部分群Hがあったときに、定義されるG/Hというのは
x∈Gについて考えられる全ての集合xHについての、
「集合xHたちの成す集合」ということでしょうか?

それからxHは必ずしも群をなさないですよね。

xによる剰余類という言い方をしますが、まずGの部分集合Hがあって、
初めてxHという剰余類が考えられるということでよろしいのでしょうか?

使っている教科書は
日本評論者の「ガロア理論講義」(足立恒雄)です。

よろしくお願いします。
>>885
正規部分群じゃないときは変なのができたりするな。
「xによる剰余類」ってのがどういう時に使われる言葉か知らないけども
Hがはっきりしてるときだな。
xを法とする剰余類という意味の時もあるな。
>885
横田「群論入門」でも読むべし
888881:03/08/21 19:33
>>884
解答の流れも教えていただきたいのですが…
よろしくお願いします。
>>885
キミのレベルでいきなりその本はつらいと思う
890132人目の素数さん:03/08/21 19:42
近所に教えてくれる人がいないのでココに質問します
次の2直線の作る角を求めよ
6x=2y=3z、x=1-y=z-1

あと
△ABCにおいて、辺ABの中点をD、辺ACを2:1に内分する
点をEとする。線分CD、BEの交点をPとし、
ベクトルAPをベクトルa、ベクトルbを使って表せ。
>>888,889
群論入門。読んでみます。>>885で書いた理解で、その後が(自分のおつむの中では)
矛盾なく進んでいるのですが、自分の理解がまちがっていたら・・・
という恐怖感にさいなまれていたための質問でした。
剰余類がでてきてから、今自分が読んでいるところ(同型定理)までが無駄になっていないか
どうかということを教えていただければ幸いです。
892884:03/08/21 20:13
>>888
辺の長さをa,b,cとする
(1)a≧b≧cとする(重複した解答がでないために)
(2)三角形の定理2辺の和は他の1辺より長いことを利用
例えば b+c>a
(3)a+b+c=24
これらを使って求める
>>885
>集合xHたちの成す集合
それでよいと思います。
>xによる剰余類
普通はxが代表する(に代表される)剰余類とかxの属する剰余類などと言うと思います。
前者の理由はx≠yであってもxH=yHとなる場合もあるので、G/Hにおける
剰余類の表し方は一意的ではないからです。
後者に関してはG/Hの元はGの部分集合ですが、x∈G を含むG/Hの元は
唯一つであり、それはxHだからです。
>>885さんも言われるように「**による剰余類」と言うと、「**を法とする剰余類」とることもありますから。
x∈Gが明言されていないと誤解される恐れがあるのではないでしょうか。

「詳解 代数入門」彌永昌吉・有馬哲・浅枝陽(東京図書)
個人的にはこの本など読みやすいと思われます。

>>893
ありがとうございました。
疑問がかなりクリアになりました。
これからアーベル群に突入します。
895893:03/08/21 20:26
> 885さんも言われるように
↑887さんの間違いでした。
>>890
この直線は交わってないですね・・・。
2直線が交わっていないときもそれらの成す角を定義するなら、
それぞれの直線の方向ベクトル同しの成す角ということでしょう。
そこで具体的にそれぞれの直線の方向ベクトルを取り。
それらの成す角をθ(0≦θ<π)として内積を2通りの仕方で求めてみてはどうでしょう。

>△ABC・・・
ベクトルa、ベクトルbの定義がないですね。
>>885
Gを群、Hをその部分群とする。
xH:={xh|h∈H}とおくと関係R(x,y) xH=yHはGの上の二項関係になる。
G/H:={xH|x∈G}としGのHによる剰余類と呼ぶ。
この場合xHのこともxの剰余類と呼ぶ。類という言葉を異なる2つの階層
で使っている。G/Hは2^Gに対して、xHはGの元に関して。
だからxHのことをxの(Hによる)剰余類と表現することは間違いではない。
ただ個人的にはxHのことをxの類とでも呼ぶべきで、剰余類はG/Hのことを
言うほうがいいような気がする。(最も無難なのはいちいちHを付けて
xHと呼ぶこと)
>>897
剰余類という言葉はG/Hの元のことを指して言うのであって
G/Hは剰余類集合と呼ぶべきものですよ。
これが群構造をもつとき剰余類群というのです。
2^Gと書かれても分からない人もいると思われ。
900132人目の素数さん:03/08/21 22:19
正8角形を異なる8色の絵の具で塗り分ける方法は何通りか、答えよ。
901132人目の素数さん:03/08/21 22:20
七本の棒があります。それぞれの両端に玉が計14個ついています。
ここから四個選ぶとき二個だけが同じ棒にある確率は??
わぁからん〜!
902132人目の素数さん:03/08/21 22:35
>>212
マルチポストは(・A・)イクナイ!!
元レスに答えが来てるぞ。

◆ わからない問題はここに書いてね 122 ◆
903865:03/08/21 22:46
>>867さん、ありがとうです!
>902
僕212ですけど、なにか落ち度ありました?
>>904
誤爆した。>>212>>901
スマソ。
一桁もあってない
>>902
どんな誤爆だYO!
関数f(t)が周期Tの周期関数とするとき
f(t)=(1/2)*A0+納n=1,∞][An*cos(nwt)+Bn*sin(nwt)]     
w=2π/T と表すことができ、このような級数をフーリェ級数と呼ぶ。と本にかいてありますが
何故このように表せるのか教えてください
>>908
必ず表せるとは限らないよ
でも表せたらうれしいでしょ
△ABCで、∠Bの二等分線と∠Cの外角∠ACDの二等分線の交点をEとするとき
∠E=1/2∠Aであることを証明しなさい
っていう問題です。中三レベルですが、よろしくおながいします
∠A=2α
∠B=2β
として
ACとBEの交点をFとおき
△ABFと△EFCの内角の和を考える
912910:03/08/22 15:20
難しい……。図形苦手なので、よくわかりません。説明いただいたのに申し訳ございません
よろしければ、もう少し詳しく教えていただけませんか?
∠A=2α
∠B=2β
とおくと∠ACDは 2α+2β
よって∠FCE=α+β
おお!できました!どうもありがとうございました!
>>907
別スレ(【質問受付】高校数学【息抜き】)の212にも、>>901
マルチしてたらしい。
916132人目の素数さん:03/08/23 16:04
(1)平面閉曲線の曲率の絶対値が正の定数1/R以下ならば、
その閉曲線の長さは2πR以上であることを証明せよ。

(2)曲線の絶対値が正の定数1/R以下であるような平面閉曲線
のうちで、長さ最小のものを求めよ。

(2)は、なんとなく、きれいな円(楕円とかじゃない円)
が答えだろうなとは予測はつくのですが、数学的にそれを
証明できません。分かる方いらっしゃいますか?
(1)は、閉曲線のどの点においても曲率が1/Rで一定だと仮定す
ると、曲率一定ってことは、この閉曲線は円だと思うので、
確かに円周の長さは2πRになりますが、もっと数学的に
きちんと証明したいです。
よろしくお願いします。ちなみに、曲率って、
閉曲線ベクトルがp(t) = (x(t),y(t))のときは、
{x'(t)*y''(t) - x''(t)*y'(t)} / [ {x'(t)}^2 + {y'(t)}^2 ]^(3/2)
ですよね。これを(1)で利用できないかなあと
思ったんですけど・・・
あと二分。
>>917
もうとっくに二分たったけど、何があと二分なの?
できないが1003になった。
920132人目の素数さん:03/08/23 20:58
980 名前:132人目の素数さん :03/08/23 20:17
>>976
「sinの微分がcosになる」という法則を使った時点で君は
「x>0のとき、x>sinx、x<0のとき、x<sinx」を認めたことになる。
逆に「sinの微分がcosになる」という法則を使って
「x>0のとき、x>sinx、x<0のとき、x<sinx」を証明しても、
それはただの循環論法だから証明として認められない。


lim=lim{2cos(x+凅/2)sin凅/2}/凅=limcos(x+凅/2)[(sin凅/2)/凅/2]=cosx
凅→0
なんですが、どうして上の人はこう言ってるの?

と思ったら理解できました。
limsinx/xのところですね
凅→0

sinx-x=0を解け、の出題者はDQNということで。
921さくらスレ980:03/08/23 21:02
>>920
三角の微分法を用いた解答に○を付けたらそいつはDQN。
しかし始めから狙っていたという可能性は否定できない。
>>921
でも、やってみてつまらなかったです。
x>0のとき、
(1*sinx)/2<(1^2)x/2より
limsinx/x=1
凅→+0
923132人目の素数さん:03/08/23 21:13
数学の節穴が塞げたから、それだけが収穫です。
あの灘生に理解できるかどうかわかりませんがw
924132人目の素数さん:03/08/23 21:15
次の曲線および直線で囲まれた図形の面積を求めよ。
曲線x=1+1/t,y=t-1/t (1≦t≦2)とx軸と直線x=5/2

という問題がわかりません。直線x=5/2をどう使うのかがわからないのです。
ぜひ教えてください。
>>924
概形書いて判断汁
>>924
x=3/2の間違い。
927質問者:03/08/23 21:38
r>0とし、
S1(r) = {(x,y)∈R^2 | x^2 + y^2 = r^2},
S2(r) = {(x,y,z)∈R^3 | x^2+y^2+z^2 = r^2}
とおく。このとき、次を示せ。

(1)(-r,r)×S1(r)の元(k,t)に対して、
ψ(k,t) = (k,sqrt(r^2 - k^2)*t/r)によってψ(k,t)
を定めると、ψは(-r,r)×S1(r)からS2(r)への
写像になる。

(2)(1)で定めた写像ψ:(-r,r)×S1(r)→S2(r)
は面積を保つ。


そもそも2次元のベクトル同士ってどうやって外積計算
するんですか?
3次元なら、(a,b,c)×(d,e,f) = (bf-ce,-af+cd,ae-bd)
ってなるのは分かるけど・・・
しかも、ψ(k,t) = (k,sqrt(r^2 - k^2)*t/r)って、ψ
の像も2次元ベクトルだから、これS2(r)の部分集合になる
とは思えないんだけど・・・(汗)

どなたかよろしくお願いします。
「8人A〜Hでトーナメントを行うとき異なる組み合わせは何通りあるか」
と言う問題で、解答の方には7*5*3*3=315(通り)となっていますが、
何故こうなるかわかりません。
8C4が7*5*3なのでそれと何らかの関係があるとは思うんですが。
>>927
(a,(b,c))=(a,b,c)としているんでしょ。
>>928
その模範解答の作者がどうやってその式をだしたのかはさっぱりわからないけど
トーナメントのくみあわせなら8!/2^7ででるよ。
>>930
ありがとうございます。何とか直観的に理解しました。
行列論で
   Col(AB) <= Col B  ただし Col A は行列 A の一次独立な列ベクトルの最大個数
とかいう定理が出てきたのですが、証明が理解できませんでした。

行列論の証明って理解しなければいけないんですか?
「結果だけ覚えて計算はできる」を目指しちゃダメですか?
933132人目の素数さん:03/08/23 22:18
>>920って何?電波のお手本?
934福田和也:03/08/23 22:37
以前にここで似たようなことを聞いたんだが、

f:R―→R

が連続であるとは、直観的にいって、連続な区間の像が必ず連続になる、
と言うことで正しかった。では、位相空間において
f:X―→Y
が連続であるとは、

任意の連結なA⊂Xに対してf(A)が連結である

ということなのか。あくまで直観的な話として。
フレシェの抽象空間論買いたいけど、金がないからここで聞いた。
気になって眠れん。
>>932
君の専攻による。てか、ただの rank 計算ちゃうん?
>>934
直観がどう云うものかは知らんが、君の直感はまちがっとる。
素直にYのopen setを引き戻すとXでもopen
でいいと思うけど
>>935
工学系志望です。
Rank A を定義する準備として Col A と Row A を定義して、
その次の行にいきなり Col(AB) <= Col B の定理が書かれてますた。
証明は意味ぷーです・・・。
939132人目の素数さん:03/08/23 22:57
Q(m)=〔{(2m)!}^2*(3/64)^m〕/{(m!)^4}
lim[m→∞]Q(m)
がわかりません
おしえてください
940132人目の素数さん:03/08/23 22:58
Σ(シグマ)の下に不等式(i1<・・・・<ir等)があるのは
どういう意味でしょうか。
普通は下にi=0、上にnってあって、i=0からnまで足すってことですよね。
しょうもない質問ですみません。
工学部で数学はあくまでもツールとして使うとしたって
それくらいは理解できないとヤバイんちゃう
まあわかんないときは2×2で具体的にいじってみて
本に書いてあることをイメージするようにしたらよろし
>>938
基底がどれだけつぶれずに残るかってことだから、自明っちゃあ自明なわけで。
>>940
君が普通と思ってることのほうが実は特殊。
Σ{x ∈ X | P(x)} で、P(x) を満たすような X の元の和って風にも
書けるし、コレを普通は Σ_[P(x)] x なんて風に書いてるわけだ。
944943:03/08/23 23:09
{a_k}_{k=1}^{n} という数列に関する和のことを
Σ_{k=1}^{n} a_k と書いているといった風に考えた方が良いかもしれない。

有限数列なら {(a_k)| k=1,...,n} とも表せるし,
その和を Σ_{k=1,...,n} a_k 等と書くこともある。

Σ の下(または上)の文字ってのは、どれがどのような条件を満たすように
動くときの和を考えているのかが書いてあるって事。
>>939
a,bが正の数のとき
((2m)!/(m!)^2)a^mb^m≦(a+b)^(2m)。
a=b=1として
(2m)!/(m!)^2≦(1+1)^(2m)=2^(2m)。
946940:03/08/23 23:23
>>943
ありがとうございました。
947927:03/08/24 01:15
>>929
じゃあ、この問題の例だと、どんな風になりますか?
948132人目の素数さん:03/08/24 01:38
1/200で第一抽選にあたりその1/3で大当たりってのと
1/600でいきなり大当たりとなるくじ引きとはどっちが
波が荒くなるですか?
>>948
波って何?
950132人目の素数さん:03/08/24 01:39
(a,b,c)を単位ベクトルとする。次の積分を求めよ。

∫_{x^2+y^2+z^2<=1} |ax+by+cz|dxdydz

極座標に変換しましたが、当方、絶対値の中が上手に整理できないため
符号がわからず絶対値を外せません。アドバイス下さい。おねがします。
>>927
問題文の × は外積じゃなくて、直積のこと。
(-r,r)×S1(r)は 1次元開区間と円周の直積
952132人目の素数さん:03/08/24 01:46
>>949
たとえば大当たりした時に+100点などとしたときにグラフにしたら
グラフが荒くなるかっていうかそーいうことです。

一般に分母の小さい物ほど少ない試行で確率に収束しやすくなると
思うんで1/200のほうがグラフが穏やかになりそうなんですが。
>>950
よくしらんけど、その単位ベクトルを(1,0,0)に回転したら?
>>952
いやだからさ、波ってのをちゃんと定義しれ。
定量的に比べられるように。
>>954
では>>948はどちらのほうが収束が早くなりますか?

>>948を何回目で当たったかを数万回繰り返しグラフにプロットしてくと
どっちほうが点が分散してるか(点が分散してる=波が荒い)

数学初心者なので意味不明な事いってすいません。
956927:03/08/24 02:21
>>951

なんだ、そういうことか。ありがとう。
出題者、ちゃんと断り書きしてほしいよねw
(1)は簡単に解けました。

(2)の面積を保つってのは、(-r,r)×S1(r)という
空間上に張った平面を、S2(r)の空間に写像すると、
面積は変わらない、という意味ですよね?

なんか、自明な気がするんですが・・・
3次元から3次元だし・・・どう証明するんだ・・・?
957948:03/08/24 02:31
最後に>>948
数学的にまったく同一のものと捉えて問題ないのですか?
分散とか確率分布とか標準偏差とかいろいろ用語がありますが
(用語の意味は良く知りません)
それらは全部同一のものになりますか?

これだけでも教えてくださいお願いします。
>1/200で第一抽選にあたりその1/3で大当たり
意味が分からない
959948:03/08/24 04:04
>>958
前者はAの抽選箱には200個ボールがあってBの抽選箱には3個のボール。
Aの抽選箱でNO1とかかれたボールをひくとBの抽選箱のほうを始めてひける。
BでもNO1をひけば大当たりとします。ってことです。

後者はCの抽選箱には600個のボールがあります。
1番を引けば大当たりです。

確率的には両者とも1/600だと思うですが数学的にどっか違いはあるのですか?
AとBの抽選をセットで一回と数えるなら
一回あたりの確率はCと同じ
別々にカウントすれば当然そうはならない。
961132人目の素数さん:03/08/24 04:31
台のスペックが載ってるサイトを聞いた方が早い
962948:03/08/24 04:38
>>960
やはり一緒ですよね。どうもでした。

(Aの抽選で外れたらBにはいきません。ハズレが1回とカウント)
>>927
>そもそも2次元のベクトル同士ってどうやって外積計算
>するんですか?
そもそも問題文のどこに2次元のベクトル同士の
積らしき演算が書いてあるんですか?
>>964
>S1(r) = {(x,y)∈R^2 | x^2 + y^2 = r^2}
966132人目の素数さん:03/08/24 10:35
以下のような問題(確率)で、無記名投票の場合と記名投票の場合で
確率が異なるというのが直感的に分かりません。
無記名だろうが記名だろうが確率は同じになると思うのですが・・・
(問題)3人の候補者がいる選挙で3n人の投票者が1人1票を投票する場合、
3人の候補者が全てn票づつ得票する確率。
>>966
同じだろ
968966:03/08/24 10:45
>>967
いえ、無記名の場合
2/((3n+2)*(3n+1))
記名の場合
(3n)!/(3^(3n)*(n!)^3)
になります。
969質問:03/08/24 11:28
他のスレで全然相手にしてくれなかったので、ここで質問させていただきます。

平面ベクトル

平行四辺形ABCDの辺BCを3:1に内分する点E、辺CDを2:1に内分する点F、
辺DAを3:1に内分する点G、線分BFとEGの交点をPとするとき、
BP:PF、EP:PGの値を求めよ。
ただしV(AB)=V(a) V(AD)=V(b)とする。

計算した結果
V(AP)=(3/4)*{( 5V(a)+4V(b) )/9}

  となったんですが、答えの「値」が求められません。

一応答えは BP:PF=9:7   EP:PG=3:5
となっています。


AF↑とか求める
971Nanashi_et_al.:03/08/24 11:47
あなるふぁっく??
972696:03/08/24 11:50
>>970
求めました。
>>969
AP↑=sAB↑+(1-s)AFというふうに
AP↑をAB↑,AF↑で表そう
974696:03/08/24 12:00
>>973
それでもとめた結果、
V(AP)=(3/4)*{( 5V(a)+4V(b) )/9}
となったんです。
975132人目の素数さん:03/08/24 12:01
クオータニオン(ハミルトン四元数)の計算について聞きたいことがあるのですが
axb + cxd + ... e = 0
といった形式の一次方程式を解く必要に迫られて、色々やっていたのですが
結局、クオータニオンをバラバラに分解してから連立方程式で解くといった方法しか自力ではできませんでした。
クオータニオンのまま計算することもできそうな気がするのですが、なにか良い方法はあるでしょうか?
ついでに2次程式の解法とかについても教えてもらえるとありがたいです。

あと、クオータニオンの微分とかを調べたいのですが、ちょっとやってみると交換法則が使えなくて実数の微分のようには行かないようです。
この辺のことについて詳しい本があったら紹介してください。

ちなみにクオータニオンはまったくの素人です、ゲームのプログラムで必要になっていて困っています。
976132人目の素数さん:03/08/24 12:09
>>968
その理由っていうか計算過程もかいておくれ
>>964-965
どちらも「2次元ベクトル同士の演算」ではないじゃん

978トップエリート街道さん ◆BIG6e4aEMg :03/08/24 12:31
>>975
面白そうですね。Quaternion C++ Class でググルと参考になるかも
>>977
だから違うって書いてるんじゃん。
980132人目の素数さん:03/08/24 12:37
【芸能】雨上がり決死隊がホークスに抗議!「表現の自由の侵害」
http://tv3.2ch.net/test/read.cgi/geinin/1060061213/600
 フジテレビのバラエティー番組「ワンナイ」でダイエーホークスの王貞治監督を侮辱したコントを放送し、王監督およびダイエー球団から抗議と謝罪
を求められていた件について23日夜、雨上がり決死隊の宮迫博之さんが初めて公式に会見した。
 宮迫さんは会見で、「王監督が怒ってしまわれるのは当然です。しかし、我々芸人は常にリスクを背負いながら笑いをとっていかなければならない。

それを承知の上でやっているんです。これくらいのコントで過敏に反応されると、これから非常にやりにくくなる。だからこれからも笑いの方針を変え
るつもりはない。」と、間接的に抗議した。
 先日の「ワンナイ」でフジテレビの須田アナウンサーが謝罪して間もないこの発言は再び物議をかもしそうだ。
>>974
この問題なら普通は
(以下、アルファベット一文字でAを始点としたベクトルを表すとする。例:F=AF↑)
P=xF+(1-x)B
P=yG+(1-y)E
0<x,y<1
とでも置いて、両辺の右辺をV(a)とV(b)で表して
x,yの方程式を解く

今の場合はx=9/16,y=3/8となるから
確かにBP:PF=9:7,EP:PG=3:5となってる
982966:03/08/24 12:46
>>976
『問題』3人の候補者がいる選挙で3n人の投票者が1人1票を投票する場合、
3人の候補者が全てn票づつ得票する確率。

無記名の場合、票の分かれ方は
区別の無い3n個の票を区別のある3つの箱に入れる場合の数なので、
H[3,3n]通りで、このうち3人が皆nを得るのは1通りなので確率は
1/H[3,3n]=2/((3n+2)*(3n+1)) となります。
記名の場合、票の分かれ方が3^(3n)通りで、皆の得票が各n票になるのは
第1の候補者が3n人からn人の票を得て、第2の候補者が残る2n人から
n人の票を得て、第三の候補が残るn人の票を得る場合なので、
その総数=C[3n,n]*C[2n,n]*C[n,n] となり、
確率は 総数/(3^(3n)=(3n)!/(3^(3n)*(n!)^3) となります。
 解答の考え方はわかるのですが、それが一致しないのが釈然としないのです
長くなり申し訳ありません。
983132人目の素数さん:03/08/24 12:51
f(z)=∫[0≦x≦∞] z^2/(z^2+1)^2 dz

を留数定理を使っても求めよという問題なんですが
特異点はz=i , -i (どちらも2位の極)領域C内にあるのはz=iの時に留数は
Res[f,i]=(-1+i)/4
f(z)=2πi*Res[f,i]=-π(i+1)/2と計算するとなったんですが合っていますでしょうか?
よろしくお願いします
>>969>>973
a,bはベクトル表記とする。
 V(AC)=a+b, V(AE)=a+0.75b, , V(AF)=0.5a+b, V(AG)=0.25b
0<s,t<1なるs,tが存在し、
 V(AP)=sV(AE)+(1−s)V(AG)=sa+(0.25+0.5s)b
     =(1−t)V(AB)+tV(AF)=(1−0.5t)a+tb
これを解いて、s=0.7,t=0.6,V(AP)=0.7a+0.6b

 V(BP)=V(AP)−V(AB)=−0.3a+0.6b
 V(PF)=V(AF)−V(AP)=−0.2a+0.4b
 ∴ |BP|:|FP|=3:2
同様に
 V(EP)=V(AP)−V(AE)=−0.3a−0.15b
 V(PG)=V(AG)−V(AP)=−0.7a−0.35b
 ∴ |EP|:|PG|=3:7

これって、答えと違うんだが、計算は間違っていないようだし…
どこが違っているんだか??
>>982
試行は立候補の誰かに投票する(立候補は均等な確率をもつ)ことを
3n人の人が行う(それぞれ独立)。だろうから記名だろうが無記名だろうが
3^(3n)通りの等確率な票の組み合わせがある。よってその前半は間違い。
特別な投票法を実施するというなら別だが、それが指定されているならば
それも明記しなければ問題にならない。
>>983
2位の極なら、留数が0ではない?
987トップエリート街道さん ◆BIG6e4aEMg :03/08/24 13:26
Remark: even solving linear equations is rather more complicated than shown
here. The most general linear equation would be Sum( a_n X b_n ) = c for
fixed quaternions {a_n}, {b_n}, and c. Probably the most efficient way to
solve these would be to view X as an unknown vector in R^4, expand out
the individual summands a_n X b_n as linear transformations M_n(X), then
add. These equations may not have solutions, nor need they be unique. For
example, iX-Xi=1 has no solutions, and iX-Xi=0 is true for all X in the
linear span of 1 and i. In the particular case shown first, uniqueness
is rather easy to check: if X1, X2 are solutions of aX+Xb+c = 0, then
their difference X3=X1-X2 satisfies aX=-Xb. Assuming X1, X2 are distinct,
X3 is invertible, giving X3^(-1) a X3 = (-b). This is a contradiction for
most pairs {a, b}.

4元数の方程式は難しいみたいですね
>>986
2位の曲でもローラン展開の-1次の項が非0なら留数≠0よ。
新スレ勃てたんだね。おめでとう!!

くだらねぇ問題はここへ書け ver.3.14(23桁略)8327
http://science.2ch.net/test/read.cgi/math/1061701221/
990 ◆Ea.3.14dog :03/08/24 14:21
くだらねぇ問題はここへ書け ver.3.14(23桁略)8327
http://science.2ch.net/test/read.cgi/math/1061701221/
ところで、↑こういうカッコいいトリップって、どうやって捜すの?
トリップの作り方って、解明されているの、それとも、試行錯誤?
>>991
総当りで検索するソフトがある。
http://tripsage.hp.infoseek.co.jp/
993132人目の素数さん:03/08/24 14:36
>>987
英語苦手なので読めていないかも知れませんが、結局僕のやった方法が一番効率的なんですかね?
とりあえずやってみた感じだと、複素数だと共役が重要ですが、クオータニオンには、これに加えて何かスパイス的な変換が必要かなと感じています。
それをどのように組んでみるかと色々と試行しているのですが、なかなか上手く行きません。
上手く行きそうでいかないのが四元数の魅力ですが、こんなのいつまでもやっていたら怒られる〜
994132人目の素数さん:03/08/24 15:51
>>992
どうもありがとう!
>>984
V(AF)=0.5a+b←これ間違い
V(AF)=(1/3)a+b←こう
>>995
ありがとう。
問題の読み間違いか!情けない…
997
998  
999    
余裕で1000get  
10011001
このスレッドは1000を超えました。
もう書けないので、新しいスレッドを立ててくださいです。。。